BLY-302 Ch. 4 & Ch. 5

Ace your homework & exams now with Quizwiz!

Epistasis:

one gene masks the effect of another gene.

A trait, such as height, has high heritability because much of the variation between individuals is the result of genetic variation. However, not all of the variation for height in a population can be attributed to genetic variation alone. Why does genetic variation not always determine the differences in a given trait between individuals? A. Genetic drift changes the frequency of genes within a population. B. Environmental influences can change gene expression. C. Random mutations lead to the generation of new alleles. D. Independent assortment leads to new combinations of genes.

*B. Environmental influences can change gene expression.*

In some dogs, the long ear phenotype is an autosomal dominant trait. Two dogs mate and produce a litter in which 75% of the puppies have long ears. A third of the long‑eared puppies in this litter are known to be phenocopies. What are the most likely genotypes of the two parents of this litter? A. One parent is homozygous for short ears, and the other is homozygous for long ears. B. Both parents are homozygous for short ears. C. One parent is heterozygous, and the other is homozygous for short ears. D. Both parents are heterozygous. E. One parent is heterozygous, and the other is homozygous for long ears.

*C. One parent is heterozygous, and the other is homozygous for short ears.*

In 1958, two scientists, R.M. Cooper and J.P. Zubek, conducted an experiment using rats in a maze. They took a group of rats and put them in a maze, recording which were able to solve the maze the quickest. The faster rats were bred together and the slower rats were bred together for a few generations until there were two distinct groups of rats, the bright rats and the dull rats, respectively. Next, groups of the dull and bright rats were raised in different environments. An enriched environment contained toys and colors and a depressing environment was simply a cage with no toys or colors. The different experimental groups were (bright, enriched), (bright, depressing), (dull, enriched), (dull, depressing). After being raised in different environments, the rats were challenged with the maze again yielding the results indicated in the table, where changes were based on the bright and dull rats' initial performance. Chart layout: [Bright rats; Enriched environment; Depressing environment] [Dull rats; Good; No change] What genetic phenomena best explains the change in performance of the bright rats raised in a depressing environment? A. A small number of genes are likely attributed to the rats' performance in the maze, and only the individual rats that inherit a dominant allele will perform well. B. Performance is attributed to only the genes each rat group inherited from their parents. C. Performance can be attributed to the interaction between the rats' genes and the environment in which they were raised. D. Only the environment contributes the greatest to the rats' performance in the maze; genetics does not affect the rats' performance.

*C. Performance can be attributed to the interaction between the rats' genes and the environment in which they were raised.*

The bicoid mutation (bcd−) in fruit flies is inherited as a maternal effect recessive allele. What is the expected ratio of phenotypes in the offspring of a cross between a bcd+/bcd− female and a bcd+/bcd− male? A. 1 normal:1 mutant B. 3 normal:1 mutant C. 3 mutant:1 normal D. All normal E. All mutant

*D. All normal.* Due to the maternal effect, because the genotype of the mother is normal, then the phenotype of her progeny is also normal.

Penile hypospadias, a birth defect in male humans in which the urethra opens on the shaft instead of at the tip of the penis, results from an autosomal dominant gene in some families. Females who carry the gene show no effects. What type of trait is this birth defect an example of? A. a Y‑linked trait because the defect occurs only in males B. a genetic maternal effect because the presence of the trait depends upon the genotype of the mother C. a sex‑influenced trait because the defect occurs only in males and the gene involved is autosomal D. a sex‑limited trait because the defect occurs only in males and the gene involved is autosomal E. an X‑linked trait because the defect occurs more often in males than in females

*D. a sex‑limited trait because the defect occurs only in males and the gene involved is autosomal*

Leigh's disease is a mitochondrially inherited disease with symptoms that include seizures, fatigue, impaired reflexes, breathing problems, and ataxia. The pedigree shows the presence of Leigh's disease in three generations. Individual 4 had all sons, and individual 6 had all daughters. Which of the individuals indicated are affected by Leigh's disease? [Place the correct symbols on the pedigree showing affected and unaffected individuals.]

*image*

Gene interaction:

- Effects of genes at one locus depend on the presence of genes at other loci - Gene interaction that produces novel phenotypes - Gene interaction with epistasis: one gene masks the effect of another gene

The Siamese cat breed has a light‑colored body and dark‑colored head, tail, and feet, all of which are called points. The Burmese cat breed has a dark‑colored body and points that are almost the same color as the body. Crossing Siamese with Burmese can produce the Tonkinese cat breed, which has coat and point color that is intermediate to the Siamese and Burmese parents. In all three breeds, a temperature‑sensitive enzyme is responsible for the dark coloration of the points. However, it is active only at the extremes of the body where it is coolest. Select the statements that are true regarding the evidence of incomplete dominance and incomplete penetrance in Siamese, Burmese, and Tonkinese breeds. [Select all that apply.] - Intermediate point color in Tonkinese demonstrates incomplete dominance. - Burmese with dark color at the head, tail, and feet demonstrates incomplete dominance. - Intermediate point color in Tonkinese demonstrates incomplete penetrance. - Development of points in Tonkinese demonstrates incomplete penetrance.

- Intermediate point color in Tonkinese demonstrates incomplete dominance. - Development of points in Tonkinese demonstrates incomplete penetrance.

Y-linked Characteristics:

- Only present in males - All male offspring will exhibit the trait - Y chromosome lost DNA over time - Important for sex determination in SRY

Early twentieth century Russia was in great turmoil as the Bolshevik Revolution ended 300 years of monarchical rule. Tsar Nicholas Romanov, his wife, Tsarina Alexandra, and their five children were placed under house arrest, but disappeared in the summer of 1918. Rumors abounded about their possible execution or escape from Russia. Two years later, a young woman named Anna Anderson claimed that she was Anastasia, the youngest daughter of Tsar Nicholas II. There was much controversy surrounding Anna's claim. Some believed she was a fraud, others that she was indeed the lost Princess Anastasia. Anna remained steadfast in her claim until her death in 1984. In 1991, the remains of nine skeletons, five male and four female, were exhumed from a shallow grave east of Moscow. Evidence from nuclear DNA showed that three of the young women were related and that one of the men and one of the women were their parents. Further evidence from mitochondrial DNA (mtDNA) showed that one of the women could be positively identified as Tsarina Alexandra and that one of the men was indeed Tsar Nicholas II. The other three women had mtDNA that matched that of the Tsarina's and were identified as three of the Tsar's children. Anastasia and her younger brother, Alexei, were not among those found in the grave. This led to further speculation about Anastasia's possible escape from Russia after her parents were killed. In 2007, an additional grave was found near that exhumed in 1991. It contained the remains of a teenage girl and boy. These remains were mtDNA tested, as well as samples from Anna Anderson, uncovered from hospital storage long after her death. Analysis of the mtDNA supported the hypothesis that Princess Anastasia had been killed with her family in 1918. Which statements support this hypothesis? [Select all that apply.] - The mtDNA from the remains of the teenage girl matched that of Tsarina Alexandra. - The mtDNA of the teenage girl and boy matched each other. - The mtDNA from the remains of the teenage girl matched that of Tsar Nicholas II. - Tsarina Alexandra's and Anna Anderson's mtDNA matched one another. - The mtDNA of Anna Anderson bore no resemblance to that of Tsarina Alexandra.

- The mtDNA from the remains of the teenage girl matched that of Tsarina Alexandra. - The mtDNA of Anna Anderson bore no resemblance to that of Tsarina Alexandra.

Match each phenotype description to its corresponding sex chromosome genotype in humans. - XO with SRY on an autosome? - XO? - XXX? - XXY? - XYY? Answer bank: A. female with Turner syndrome B. phenotypically male with sterility and hypogonadism C. phenotypically female with some abnormalities and overexpression of X chromosome genes D. phenotypically male with an increase in average stature E. phenotypically male but karyotype indicates presence of only X chromosome

- XO with SRY on an autosome? E. phenotypically male but karyotype indicates presence of only X chromosome - XO? A. female with Turner syndrome - XXX? C. phenotypically female with some abnormalities and overexpression of X chromosome genes - XXY? B. phenotypically male with sterility and hypogonadism - XYY? D. phenotypically male with an increase in average stature

In eukaryotes, extranuclear inheritance occurs when genetic information is transmitted by mechanisms other than through nuclear DNA. Chloroplast DNA (cpDNA) is an example of one mechanism by which extranuclear inheritance can occur. Select the statements that correctly describe cpDNA. - cpDNA is typically inherited from both parents. - Chloroplast chromosome size and gene content are identical in all organisms. - cpDNA organization is more similar to that of prokaryotes than eukaryotes. - Chloroplast chromosomes contain genes that are involved in photophosphorylation.

- cpDNA organization is more similar to that of prokaryotes than eukaryotes. - Chloroplast chromosomes contain genes that are involved in photophosphorylation.

Both genetic and nongenetic factors cause the congenital skeletal abnormality known as clubfoot, which occurs with a worldwide incidence of about 1 in 1000 births. Gurnett et al. identified a family in which clubfoot occurred as an autosomal dominant trait due to a mutation in the PITXI gene. DNA testing revealed that 11 people in the family carried the PITXI mutation, but only eight of these people had clubfoot. Calculate the penetrance of the PITXI mutation in this family. - penetrance? ______ %

- penetrance: 73%

A scientist crosses the fly with vestigial wings, shown on the bottom‑left of the figure, to the fly with normal wings, shown on the right, and rear the progeny at 31°C. What percentage of offspring have vestigial wings? - percentage: _______ %

- percentage: *0%*

Assume that long earlobes in humans are an autosomal dominant trait that exhibits 30% penetrance. A person who is heterozygous for long earlobes mates with a person who is homozygous for normal earlobes. What is the probability that their first child will have long earlobes? [Use two decimal places for the answer.] - probability?

- probability: *0.15%*

Suppose that allele D produces big ears and allele d produces normal ears. A heterozygous male with the Dd genotype mates with a homozygous female with the dd genotype. Big ears is an autosomal dominant trait expressed with 50% penetrance. Determine the probability that their offspring will have big ears. Report your answer as a percentage without decimal points. - probability? ______ %

- probability: 25%

Allelic series (multiple alleles):

-for some loci, more than two alleles are present within a group of organisms-the locus has multiple alleles -there is no difference in inheritance; although there will be a greater variety of genotypes and phenotypes possible # genotypes possible= [n(n+1)/2) where n equals the number of different alleles at a locus.

Suppose that a rabbit breeder notices two individuals in a litter with large, round noses and names this trait the clown trait. He obtains another rabbit that has a long, thin nose and names it the Pinocchio trait. After breeding the rabbits with unusual nose shapes to wild‑type individuals for several generations, he decides to cross rabbits from families with a history of the clown and Pinocchio traits. The first litter from this cross results in individuals of four phenotypes. The phenotypes are clown, Pinocchio, wild‑type, and heart‑shaped noses. The pedigree illustrates the inheritance of these traits. Individuals 1 and 4 are siblings and are known to come from a true‑breeding wild‑type line. 1. What is the most likely mode of inheritance for the clown nose? A. autosomal recessive B. autosomal dominant C. X‑linked dominant D. X‑linked recessive E. Y‑linked 2. What is the most likely mode of inheritance for the Pinocchio nose? A. X‑linked dominant B. sex‑influenced C. X‑linked recessive D. autosomal recessive E. autosomal dominant A careless rabbit sitter leaves several cages open, and a female rabbit mates with an unidentified male. The breeder sends DNA samples from each of the offspring whose fathers are unknown and DNA from individuals IV‑5 through IV‑10 to a genetics lab. The lab staff test the samples for several genes with suspected effects on nose shape. The gel displays the results of genetic testing for the two polymorphic genes detected. The individuals whose father is unknown are labeled A, B, C, and D. 3. What is the phenotype of individual C? A. Pinocchio B. wild‑type C. clown D. heart‑shaped

1. *D. X-linked recessive* 2. *E. autosomal dominant* 3. *A. Pinocchio*

Suppose a species of tulip has three alleles for the gene that codes for flower color. The 𝐶𝑅 allele produces red tulips, the 𝐶𝑝 allele produces purple tulips, and the 𝐶𝑤 allele produces white tulips. 𝐶𝑅 is dominant over 𝐶𝑝 and 𝐶𝑤, and 𝐶𝑝 is dominant over 𝐶𝑤. For each cross, determine the expected phenotype ratio of offspring flower color. 1. CRCp x CpCw ---------- [Expected phenotype ratio] _________? 2. CRCw x CpCw ---------- [Expected phenotype ratio] _________? Answer bank: - 1 red : 1 purple : 0 white - 2 red : 2 purple : 0 white - 2 red : 0 purple : 2 white - 2 red : 1 purple : 1 white - 1 red : 2 purple : 1 white - 1 red : 3 purple : 0 white

1. 2 red : 2 purple : 0 white 2. - 2 red : 1 purple : 1 white

Hemophilia is called "the royal disease" because many European royal families had members with the condition. Hemophilia is a recessive, X‑linked disorder. Queen Victoria was unaffected by hemophilia, but was a carrier of the hemophilia gene (X^HX^h). Suppose Queen Victoria's husband, Prince Albert, was affected with hemophilia (X^hY). 1. What is the percent probability that a son of Queen Victoria and Prince Albert would be unaffected by hemophilia? - probability of unaffected son: ________ %? 2. What is the percent probability that a daughter or son of Queen Victoria and Prince Albert would be affected by hemophilia? - probability of affected child: ________ %? 3. What is the percent probability Queen Victoria and Prince Albert would have two affected females? - probability of two affected females: ________ %?

1. 50% 2. 50% 3. 6.25%

The given pedigree shows three generations of a family affected by a genetic disorder Z. Disorder Z is caused by mutations in a single gene, and it shows multiple symptoms such as muscle paralysis, nerve disorders, abdominal pain, and photosensitivity. Three of the symptoms are represented in the pedigree. 1. Select the mode of inheritance of disorder Z. A. autosomal dominant B. Y‑linked dominant C. X‑linked recessive D. autosomal recessive E. X‑linked dominant 2. Select the genetic phenomena displayed by the pedigree of disorder Z. [Select all that apply.] - expressivity - incomplete penetrance - genetic anticipation - pleiotropy

1. A. autosomal dominant 2. - incomplete penetrance; - pleiotropy

Suppose that in goats, an independently sorting autosomal allele that produces a bearded phenotype is dominant in males and recessive in females. The symbol Bb represents the bearded allele and B+ represents the beardless allele. The autosomal allele for a black coat (W), which also independently assorts, is dominant over the allele for white coat (w). Match each set of progeny phenotypes and proportions to the parental cross that produced that set. - B+Bb Ww male x B+Bb Ww female? - B+Bb Ww male x B+Bb ww female? - B+B+ Ww male x BbBb Ww female? - B+Bb Ww male x BbBb ww female? Answer bank: - males: all bearded, 1/2 white; females: 1/4 bearded, black - males: all bearded, 1/4 white; females: all beardless, 3/4 black - males: 1/8 beardless, white; females: 1/8 bearded, black - males: 9/16 bearded, black; females: 9/16 beardless, black

1. B+Bb Ww male x B+Bb Ww female: *- males: 9/16 bearded, black; females: 9/16 beardless, black* 2. B+Bb Ww male x B+Bb ww female: *- males: 1/8 beardless, white; females: 1/8 bearded, black* 3. B+B+ Ww male x BbBb Ww female: *- males: all bearded, 1/4 white; females: all beardless, 3/4 black* 4. B+Bb Ww male x BbBb ww female: *- males: all bearded, 1/2 white; females: 1/4 bearded, black*

A black female cat is mated with an orange male cat. They produce two tortoiseshell females, two black males, one orange female, and one tortoiseshell male, for a total of six kittens. Orange and black fur color are encoded by different alleles of the same X‑linked fur color gene. 1. Based on the phenotypes of the parents, what sex chromosomes do the orange female and the tortie male offspring have? A. The orange female has two X chromosomes, and the tortoiseshell male is YO. B. The orange female is missing one X chromosome, and the tortoiseshell male has one Y chromosome and more than one X chromosome. C. The orange female is missing one X chromosome, and the tortoiseshell male is XY. D. The orange female is XX, and the tortoiseshell male is XXY. 2. Select the most likely explanations for the sex chromosome types. [Select all that apply.] - The tortoiseshell male has only one X chromosome, and it carries the black allele. - Random X inactivation in XXY causes orange and black patches in the tortoiseshell male. - The orange female received an orange X chromosome from her father and no X chromosome from her mother. - The orange female has two X chromosomes, and the black X from her mother is inactivated.

1. B. The orange female is missing one X chromosome, and the tortoiseshell male has one Y chromosome and more than one X chromosome. 2. - Random X inactivation in XXY causes orange and black patches in the tortoiseshell male. - The orange female received an orange X chromosome from her father and no X chromosome from her mother.

Color blindness is a sex‑linked recessive trait. A female is color blind in one eye, but not both. 1. Select the explanation for this condition. A. female has XO genotype B. silencing of different X chromosomes C. all X chromosomes are inactivated 2. Is it possible for a male to have different color‑blindness phenotypes in each eye? A. yes, in an XXY male with a different active X in each eye B. yes, in an XYY male with a different active Y in each eye C. no, since trisomy of the sex chromosomes is lethal D. no, since XXY males inactivate both X chromosomes

1. B. silencing of different X chromosomes 2. A. yes, in an XXY male with a different active X in each eye

In certain salamanders, the sex of a genetic female can be altered, changing her into a functional male; these salamanders are called sex‑reversed males. When a sex‑reversed male is mated with a normal female, approximately 2/3 of the offspring are female, and 1/3 are male. 1. Given this observation, which of the statements is most likely to be true for these salamanders? A. Females are the homogametic sex. B. They exhibit hermaphroditism. C. Females are the heterogametic sex. D. They exhibit genic sex determination. E. Males are the heterogametic sex. 2. How do you explain the results of this cross? A. The female parent had two Z chromosomes. B. Both parents were ZW, and WW offspring did not survive. C. The sex‑reverse parent was ZZ, the female parent was ZW, and ZW offspring did not survive. D. The sex‑reverse male parent had two W chromosomes. Q

1. C. Females are the heterogametic sex. 2. B. Both parents were ZW, and WW offspring did not survive.

Suppose that in unicorns, two autosomal loci interact to determine the type of tail. One locus controls whether a tail is present at all. The allele for a tail, T, is dominant over the allele for no tail, t. If a unicorn has a tail, then alleles with an unknown dominance relationship at a second locus determine whether the tail is curly or straight. Define the alleles for tail texture as US for a straight tail and UC for a curly tail. Farmer Baldridge has two unicorns with curly tails. When he crosses them, one‑half of the progeny have curly tails, one‑quarter have straight tails, and one‑quarter do not have a tail. 1. What are the genotypes of the parents at the tail‑presence gene? A. Tt × tt B. TT × TT C. Tt × Tt D. TT × tt E. tt × tt 2. What are the genotypes of the parents at the tail‑texture gene? A. USUC × UCUC B. USUS × USUS C. USUC × USUC D. UCUC × UCUC E. USUS × UCUC 3. Place the unicorn genotype or genotypes below each unicorn phenotype in the pedigree. Some unicorn phenotypes may be the result of more than one genotype. - TT USUC - Tt UCUC - Tt USUS - TT USUS - tt USUC - TT UCUC - Tt USUC - tt USUS - tt UCUC

1. C. Tt x Tt 2. C. USUC x USUC 3. [top, left]: *Tt USUC* [top, right]: *Tt USUC* [bottom, left]: *TT USUC*; *Tt USUC* [bottom, middle]: *TT USUS*; *Tt USUS* [bottom, right]: *tt USUS*; *tt USUC*

Suppose a novel disease has been studied in several families and is known to be caused by a mutation in one gene. The given table shows families where the disease is present. The four rightmost columns record the number of offspring in each family. 1. Select the most likely mode of inheritance of the disease. A. autosomal recessive B. autosomal dominant C. X‑linked dominant D. X‑linked recessive 2. Select the description that best matches the female parent in family 6. A. has two disease alleles B. has no disease alleles C. has one disease allele 3. Select the description that best matches the male parent in family 3. A. has two disease alleles B. has one disease allele C. has no disease alleles

1. C. X-linked dominant 2. C. has one disease allele 3. C. has no disease alleles

Lethal allele:

1. Causes death at an early stage of development, and so some genotypes may not appear among the progeny 2. Affects the Mendelian genotypic and phenotypic ratios in progeny

In a hypothetical insect species, eye color may be purple or red and wings may be long or short. The table shows the results of two crosses where the parents were obtained from a mixed population that did not breed true. Parents: Progeny: Cross 1: red, long male × red, long female males: one‑half red, long; one‑half red, short females: all red, long Cross 2: red, short male × purple, long female males: one‑half red, long; one‑half red, short females: one‑half red eyed, long; one‑half red eyed, short 1. What is the mode of inheritance for eye color in this insect? A. The eye‑color gene is X‑linked, and purple is dominant to red. B. The eye‑color gene is X‑linked, and red is dominant to purple. C. The eye‑color gene is autosomal, and purple is dominant to red. D. The eye‑color gene is autosomal, and red is dominant to purple. 2. What is the mode of inheritance for wing size in this insect? A. The wing size gene is autosomal, and long is dominant to short. B. The wing size gene is X‑linked, and long is dominant to short. C. The wing size gene is X‑linked, and short is dominant to long. D. The wing size gene is autosomal, and short is dominant to long. 3. Assuming that true breeding lines are used, which cross would produce an F2 generation that confirms the mode of inheritance for both traits? A. red, short male × purple, short female B. red, long male × red, short female C. purple, short male × purple, short female D. red, short male × purple, long female 4. Which phenotypic ratio would you expect to see in the F2 generation of the cross you selected? A. males: 3/8 red, long; 3/8 red, short; 1/8 purple, long; 1/8 purple, short females: 3/8 red, long; 3/8 red, short; 1/8 purple, long; 1/8 purple, short B. males: 3/8 red, long; 3/8 red, short; 1/8 purple, long; 1/8 purple, short females: 3/4 red, long; 1/4 purple, long C. males: 3/4 red, long; 1/4 purple, long females: 3/8 red, long; 3/8 red, short; 1/8 purple, long; 1/8 purple, short D. males: 9/16 red, long; 3/16 red, short; 3/16 purple, long; 1/16 purple, short females: 9/16 red, long; 3/16 red, short; 3/16 purple, long; 1/16 purple, short

1. D. The eye‑color gene is autosomal, and red is dominant to purple. 2. B. The wing size gene is X‑linked, and long is dominant to short. 3. D. red, short male × purple, long female 4. B. males: 3/8 red, long; 3/8 red, short; 1/8 purple, long; 1/8 purple, short females: 3/4 red, long; 1/4 purple, long

1. Select the definition of a Barr body. A. an active X chromosome B. an active autosome C. an active Y chromosome D. an inactive X chromosome The first cloned cat, CarbonCopy (CC), was tabby, while the cat she was cloned from, Rainbow, was calico. The surrogate mother was a tabby. 2. Select the explanation that best explains why CC would never have been identical in pattern to Rainbow. A. Both X chromosomes remain actively transcribed. B. During embryogenesis, one X chromosome is randomly inactivated in each cell lineage. C. The coat color genes are maternal effect genes, so CC looked like the surrogate. D. The coat color genes are on the mitochondrial genome and inherited with the mother's oocyte.

1. D. an inactive X chromosome 2. B. During embryogenesis, one X chromosome is randomly inactivated in each cell lineage.

A homozygous variety of opium poppy (Papaver somniferum Laciniatum) with lacerate leaves was crossed with another homozygous variety with normal leaves. All the F1 had lacerate leaves (jagged‑edged leaves). Two F1 plants were crossed to produce the F2. Of the F2, 249 had lacerate leaves and 16 had normal leaves. 1. How are lacerate leaves determined in the opium poppy? A. one gene, with codominant alleles at a single locus B. two genes, with a dominant allele at both loci C. two genes, where one allele at a single locus is recessive lethal D. two genes, with a dominant allele at either or both loci 2. Give genotypes for all the plants in the P, F1, and F2 generations. Some generations will have more than one genotype. - P? - F1? - F2 (lacerate)? - F2 (normal)? Answer bank: - aabb - AaBb - AABB - A_B_ - aaB_ - A_bb

1. D. two genes, with a dominant allele at either or both loci 2. P: - AABB - aabb F1: - AaBb F2 (lacerate): - A_B_ - A_bb - aaB_ F2 (normal): - aabb

Characteristics of Dominance:

1. Dominance is the result of interactions between genes at the same locust in other words, dominance is allelic interaction 2. Only influences the way in which they are expressed as a phenotype, and doesn't alter the way in which they were inherited 3. Classification of dominance depends on the level at which the phenotype si examined

1. XX -AA; 1.0 2. XY-AA; 0.5 3. XO -AA; 0.5 4. XXY-AA; 1.0 5. XXX-AA; 1.5 6. XXXY-AA; 1.5 7. XX-AAA; 0.67 8. XO-AAA; 0.33 9. XXXX-AAA; 1.3

1. Female 2. Male 3. Male 4. Female 5. Metafemale 6. Metafemale 7. Intersex 8. Metamale 9. Metafemale

Cytoplasmic inheritance:

1. Present in males and females 2. Usually inherited from one parent; usually the maternal; never from father to offspring; cytoplasmic genes only come from one of the gametes, usually the egg 3. Reciprocal crosses give different results 4. Exhibit extensive phenotypic variation, even within a single family

Male-limited precocious puberty results from a rare autosomal allele (P) that is dominant over the allele for normal puberty (p) and is expressed only in males. A male and female that both went through normal puberty have two sons. The first son undergoes precocious puberty but the second undergoes normal puberty. 1. What is the genotype of the mother? A. PP B. Pp C. pp D. PP or Pp E. Pp or pp 2. What is the genotype of the father? A. PP B. Pp C. pp D. PP or Pp E. Pp or pp

1. [Mother:] *B. Pp* 2. [Father:] *C. pp*

Use the Punnet Squares interactive, Level 3 to conduct a cross between a man with normal vision and a woman with the gene for color blindness. 1. A man with normal vision, XB Y, and a woman who is a carrier for color blindness, XB Xb, mate. How many total phenotypes result from this cross? 2. What percentage of male offspring will be color‑blind? color‑blind male offspring? 3. If the mother were color‑blind, what percentage of male offspring would be color‑blind?

1. number of phenotypes: - 3 phenotypes 2. color-blind male offspring: - 50% 3. color-blind male offspring: - 100%

1. How many phenotypes result from a cross between the two parents in the interactive? - number of phenotypes: 2. What percentage of the offspring are predicted to have the B blood phenotype? - percentage of offspring with B blood: ______ % 3. For an individual with O type blood, what would the blood cell look like in the interactive? A. no antigens B. circle‑shaped antigens C. a different shape of antigens D. both triangle and circle‑shaped antigens E. triangle‑shaped antigens

1. number of phenotypes: 3 2. percentage of offspring with B blood: 50% 3. A. no antigens

Suppose two independently assorting genes are involved in the pathway that determines fruit color in squash. These genes interact with each other to produce the squash colors seen in the grocery store. At the first locus, the W allele codes for a dominant white phenotype, whereas the w allele codes for a colored squash. At the second locus, the allele Y codes for a dominant yellow phenotype, and the allele y codes for a recessive green phenotype. The phenotypes from the first locus will always mask the phenotype produced by the second locus if the dominant allele (W) is present at the first locus. This masking pattern is known as dominant epistasis. A dihybrid squash, Ww Yy, is selfed and produces 160 offspring. How many offspring are expected to have the white, yellow, and green phenotypes? 1. number of white offspring? 2. number of yellow offspring? 3. number of green offspring?

1. number of white offspring: *120* 2. number of yellow offspring: *30* 3. number of green offspring: *10*

If a male bird that is heterozygous for a recessive Z-linked mutation is crossed to a wild-type female, what proportion of the progeny will be mutant males? Remember that in birds, females are the heterogametic sex (ZW). A. 0% B. 100% C. 75% D. 50% E. 25%

A. 0%

A eukaryotic diploid cell from an organism with the XX-XO sex determination system has two pairs of autosomes and one X chromosome. One pair of autosomes has a heterozygous locus with alleles A and a. The other pair of autosomes has a heterozygous locus with alleles B and b. What is the probability of a gamete from this individual having this genotype: alleles a and b, without chromosome X? A. 1/8 B. 1/16 C. 1/4 D. 1/2

A. 1/8

Club foot is one of the most common congenital skeletal abnormalities, with a worldwide incidence of about 1 in 1000 births. Both genetic and nongenetic factors are thought to be responsible for club foot. C. A. Gurnett et al. (2008. American Journal of Human Genetics 83:616-622) identified a family in which club foot was inherited as an autosomal dominant trait with reduced penetrance. They discovered a mutation in the PITXI gene that caused club foot in this family. Through DNA testing, they determined that 11 people in the family carried the PITXI mutation, but only 8 of these people had club foot. What is the penetrance of the PITXI mutation in this family? A. 8/11 = 73% B. 4/11 = 36% C. 3/11 = 27% D. 1/1000 = 0.1% E. Not enough information is given.

A. 8/11 = 73%

If a homozygous plant with red peppers is crossed with a homozygous plant with cream peppers, all the F1 plants will have red peppers. When the F1 are crossed with each other, the F2 shows a ratio of: A. 9:3:3:1 B. 9:3:4 C. 3:1 D. 9:7

A. 9:3:3:1

How is dosage compensation accomplished in humans? A. All but one of the X chromosomes is inactivated. B. No obvious mechanism of dosage compensation. C. The Y chromosome has an increased level of gene expression. D. The activity of genes on the male X chromosome is doubled but not that of genes on the X chromosome of females. E. The activity of genes on half of the autosomes is doubled and the activity of genes on the X chromosomes of both males and females is decreased.

A. All but one of the X chromosomes is inactivated.

Which statements describe Y‑linked traits? [Select all that apply.] A. All male offspring of affected fathers will express the trait. B. Males have a higher probability of inheriting the trait from their mothers. C. At least 50% of the male and female offspring will inherit the trait. D. Female offspring cannot inherit the trait. E. A carrier female has a 25% chance of having a carrier daughter.

A. All male offspring of affected fathers will express the trait. D. Female offspring cannot inherit the trait.

In Drosophila, yellow body is due to an X-linked gene (Xy) that is recessive to the gene for gray body (X+). A homozygous gray female is crossed with a yellow male. What are the phenotypes of the F1 progeny? A. All the progeny are gray. B. All the progeny are yellow. C. All the females are gray and all the males are yellow D. All the females are yellow and all the males are gray. E. All the females are gray; 1/2 of the males are yellow, 1/2 are gray.

A. All the progeny are gray.

Assume that a mutation occurs in the Xist gene of a XY male that makes the gene inactive. What would you expect would be the phenotypic consequences of this mutation for the male? A. He would probably have no phenotypic consequences. B. He would probably not survive because all his X-linked genes would be turned off. C. He would not have expression of some of his autosomal genes. D. He would have enhanced expression of his Y-linked genes.

A. He would probably have no phenotypic consequences.

How does sex determination in the XX-XY system differ from sex determination in the ZZ-ZW system? A. In the XX-XY system, the male is the heterogametic sex; in the ZZ-ZW system, the female is the heterogametic sex. B. In the XX-XY system, the male is the homogametic sex; in the ZZ-ZW system, the female is the homogametic sex. C. In both the XX-XY and the ZZ-ZW systems, the male is the heterogametic sex; in the ZZ-ZW system, the female is also heterogametic. D. In the XX-XY system, the female is the homogametic sex; in the ZZ-ZW system, the male is the heterogametic sex. E. In the XX-XY system, males and females have the same total number of chromosomes; in the ZZ-ZW system, males have one less chromosome than females.

A. In the XX-XY system, the male is the heterogametic sex; in the ZZ-ZW system, the female is the heterogametic sex.

A yellow female Labrador retriever was mated with a brown male. Half of the puppies were brown, and half were yellow. Explain how the same female, when mated with a different brown male, could produce only brown offspring. A. The first male was bb Ee, and the second male was bb EE. B. The first male was bb Ee, and the second male was bb ee. C. The first male was bb Ee, and the second male was Bb Ee. D. The first male was bb EE, and the second male was bb Ee. E. The first male was Bb EE, and the second male was Bb Ee.

A. The first male was bb Ee, and the second male was bb EE.

Why are tortoiseshell cats are almost always female? A. The result is from the random inactivation of one X chromosome. B. The result is from a particular homozygous genotype. C. The allele for tortoiseshell fur color is located on a single X chromosome. D. It is the result of a particular Y-linked characteristic.

A. The result is from the random inactivation of one X chromosome.

Which of the following human genotypes is associated with Klinefelter syndrome? A. XXY B. XYY C. XXX D. XO E. XX

A. XXY

The bicoid mutation (bcd-) in fruit flies is inherited as a genetic maternal effect recessive allele. What is the expected ratio of phenotypes in the offspring of a cross between a bcd-/bcd- female and a bcd+/bcd- male? A. all mutant B. 1 normal : 1 mutant C. 3 normal : 1 mutant D. all normal

A. all mutant

Suppose researchers identified two Drosophila melanogaster mutant phenotypes. One phenotype is called thistle and the other is called coffee. In order to determine the mode of inheritance of the mutant allele responsible for each phenotype, males that have both mutant phenotypes are mated with wild‑type (wt) females to produce F1 progeny. Then, the F1 progeny are mated together and the F2 progeny are scored. What is the mode of inheritance for the genes controlling thistle and coffee? A. autosomal dominant for thistle and X‑linked recessive for coffee B. X‑linked dominant for thistle and X‑linked recessive for coffee C. autosomal recessive for thistle and autosomal recessive for coffee D. autosomal dominant for thistle and X‑linked dominant for coffee What is the genotype of the original male parent? A. heterozygous mutant at one locus and hemizygous mutant at the other locus B. homozygous mutant at both loci C. hemizygous mutant at both loci D. homozygous mutant at one locus and hemizygous mutant at the other locus

A. autosomal dominant for thistle and X‑linked recessive for coffee D. homozygous mutant at one locus and hemizygous mutant at the other locus

An X chromosome that expresses the Xist gene will likely A. become inactivated. B. remain activated. C. repress the other X chromosome in the cell. D. be degraded.

A. become inactivated.

What is the role of Mullerian-inhibiting substance in human development? A. causes the degeneration of female reproductive ducts B. causes the testes to secrete hormones C. causes the neutral gonads to develop into ovaries D. causes the degeneration of male reproductive ducts

A. causes the degeneration of female reproductive ducts

A boy has blood type MN with a genotype of LMLN. His red blood cells possess both the M antigen and the N antigen. What is the relationship between his two alleles for this gene? A. codominance B. epistasis C. complementation D. incomplete dominance E. complete dominance

A. codominance

Which type of dominance occurs when the heterozygote includes the phenotypes of both homozygotes? A. codominance B. overdominance C. incomplete dominance D. complete dominance

A. codominance

Interactions among the human ABO blood group alleles involve ________ and ________. A. codominance; complete dominance B. codominance; incomplete dominance C. complete dominance; incomplete dominance D. epistasis; complementation E. continuous variation; environmental variation

A. codominance; complete dominance

The phenomenon in which a gene's expression is determined by its parental origin is called: A. genomic imprinting. B. sex-influenced. C. sex-limited. D. maternal effect. E. paternal effect.

A. genomic imprinting.

Fur color in a species of mouse is controlled by a single gene pair. BB animals are black and bb animals are white. Bb animals have gray fur and each hair is gray. What type of interaction is being shown by the two alleles in heterozygous animals? A. incomplete dominance B. epistasis C. complementation D. codominance E. complete dominance

A. incomplete dominance

Continuous characteristics that are both polygenic and influenced by environmental factors are called which of these? A. multifactorial characteristics B. epistatic characteristics C. phenocopies D. epigenetic characteristics

A. multifactorial characteristics

In butterflies, sex is determined by the ZW sex‑determination system. Female butterflies are heterogametic and have both a Z sex chromosome and a W sex chromosome for sex determination. In contrast, male butterflies are homogametic and have two Z sex chromosomes. Select all of the relatives from which a female butterfly could have inherited her Z sex chromosome. A. paternal grandmother B. maternal grandfather C. mother D. father

A. paternal grandmother D. father

Incomplete _____ occurs when the genotype does not always produce the expected phenotype. A. penetrance B. pleiotropy C. dominance D. expressivity

A. penetrance

What is the definition of dominant epistasis? A. where only a single copy of an allele is required to inhibit the expression of an allele at a different locus B. where genes at different loci contribute to the determination of a single phenotypic characteristic C. where the presence of two recessive alleles inhibits the expression of an allele at a different locus D. the masking of genes at the same locus (allelic genes)

A. where only a single copy of an allele is required to inhibit the expression of an allele at a different locus

What is the definition of recessive epistasis? A. where the presence of two recessive alleles (the homozygous genotype) inhibits the expression of an allele at a different locus B. where genes at different loci contribute to the determination of a single phenotypic characteristic C. where two recessive alleles at either of two different loci are capable of suppressing a phenotype D. where only a single copy of an allele is required to inhibit the expression of an allele at a different locus

A. where the presence of two recessive alleles (the homozygous genotype) inhibits the expression of an allele at a different locus

An individual with Turner syndrome has how many Barr bodies? A. zero B. one C. two D. three

A. zero

An allelic series determines coat color in rabbits: C (full color), cch (chinchilla, gray color), ch (Himalayan, white with black extremities), and c (albino, all white). The C allele is dominant over all others, cch is dominant over ch and c, ch is dominant over c, and c is recessive to all other alleles. This dominance hierarchy can be summarized as C > cch > ch > c. The rabbits in the table are crossed and produce the progeny shown. Chart layout: [Cross;Parents;Offspring] [A; full color × albino; 50% full color, 50% albino] [B; Himalayan × albino; 50% Himalayan, 50% albino] [C; full color × albino; 50% full color, 50% chinchilla] [D; full color × Himalayan; 50% full color, 25% Himalayan, 25% albino] [E; full color × full color; 75% full color, 25% albino] Match the parental genotypes to the letter corresponding to the appropriate cross listed in the table. - A? - B? - C? - D? - E? Answer bank: - CC x cc - c^hc^h x cc - Cc x cc - Cc x Cc - Cc x c^hc - Cc^ch x cc - c^hc x cc

A: - Cc x cc B: - c^hc x cc C: - Cc^ch x cc D: - Cc x c^hc E: - Cc x Cc

9:3:3:1

A_B_: A_bb: aaB_: aabb; no distinguishable type of interaction

Identify the chromosomal disorders affecting the patients in the Abnormal Karyograms tab of the karyotype interactive. Place each patient according to whether his or her disorder occurs exclusively in biological males, exclusively in biological females, or in both sexes. - Affects only biological males? - Affects only biological females? - Affects both sexes? Answer bank: - patient 768 - patient 367 - patient 251 - patient 892

Affects only biological males? - patient 892 Affects only biological females? - patient 367 Affects both sexes? - patient 768 - patient 251

Calvin Bridges determined that the rare white‑eyed males produced from the mating of true‑breeding red‑eyed females with white‑eyed males resulted from a nondisjunction of X chromosomes in the female parent during meiosis. If such a mating produced 1234 red‑eyed progeny and three white‑eyed males, what is the observed rate of nondisjunction of the X chromosomes? A. 0.97% B. 0.24% C. 0.12% D. This is impossible to calculate with the given information. E. 0.49%

B. 0.24%

In some goats, the presence of horns is produced by an autosomal gene that is dominant in males and recessive in females. A horned female is crossed with a hornless male. The F1 offspring are intercrossed to produce the F2. What proportion of the F2 females will have horns? A. None B. 1/4 C. 1/2 D. 3/4 E. All

B. 1/4

The following two genotypes are crossed: Aa Bb Cc X+Xr by Aa BB cc X+Y, where a, b, and c represent alleles of autosomal genes and X+ and Xr represent X-linked alleles in an organism with XX-XY sex determination. What is the probability of obtaining genotype aa Bb Cc X+X+ in the progeny? A. 1/16 B. 1/64 C. 1/32 D. 1/128 E. 1/27

B. 1/64

In goats, a beard is produced by an autosomal allele that is dominant in males and recessive in females. We'll use the symbol Bb for the beard allele and B+ for the beardless allele. Another independently assorting autosomal allele that produces a black coat (W) is dominant over the allele for white coat (w). Give the phenotypes and their expected proportions for the cross B+Bb ww male × BbBb Ww female. A. 1/8 beardless black females, 1/8 beardless white females, 1/8 bearded black females, 1/8 bearded white females, 1/8 bearded black males, 1/8 bearded white males, 1/8 beardless black males, 1/8 beardless white males B. 1/8 beardless black females, 1/8 beardless white females, 1/8 bearded black females, 1/8 bearded white females, 1/4 bearded black males, 1/4 bearded white males C. 1/16 beardless black females, 3/16 beardless white females, 1/8 bearded black females, 1/8 bearded white females, 1/8 bearded black males, 1/8 bearded white males, 1/8 beardless black males, 1/8 beardless white males D. 1/4 beardless black females, 1/4 beardless white females, 1/4 bearded black males, 1/4 bearded white males

B. 1/8 beardless black females, 1/8 beardless white females, 1/8 bearded black females, 1/8 bearded white females, 1/4 bearded black males, 1/4 bearded white males

Red-green color blindness is an X-linked recessive trait in humans. Polydactyly (extra fingers and toes) is an autosomal dominant trait. Martha has normal fingers and toes and normal color vision. Her mother is normal in all respects, but her father is color blind and polydactylous. Bill is color blind and polydactylous. His mother has normal color vision and normal fingers and toes. If Bill and Martha marry, what proportions of children with specific phenotypes would they be expected to produce? [The answers only include the proportions of some of the possible phenotypes; other phenotypes are also expected to occur but are not included.] A. 1/4 color-blind girls with normal fingers, 1/4 boys with normal vision and polydactyly B. 1/8 color-blind girls with polydactyly, 1/8 boys with normal vision and normal fingers C. 1/3 girls with normal vision and normal fingers, 1/3 boys with normal vision and polydactyly, and 1/3 boys with normal vision and normal fingers D. 1/6 girls with normal vision and polydactyly, 1/6 boys with normal vision and polydactyly E. 1/8 color-blind girls with normal fingers, 1/4 boys with normal vision and polydactyly

B. 1/8 color-blind girls with polydactyly, 1/8 boys with normal vision and normal fingers

Red-green color blindness is an X‑linked recessive trait in humans. Polydactyly (extra fingers and toes) is an autosomal dominant trait. Martha has normal fingers and toes and normal color vision. Her mother is normal in all respects, but her father is color blind and polydactylous. Bill is color blind and polydactylous. His mother has normal color vision and normal fingers and toes. When answering the given question, consider that the answers include the proportions of only some of the possible phenotypes; other phenotypes are also expected to occur but are not included. If Bill and Martha marry, what proportions of children with specific phenotypes would they be expected to produce? A. 1/4 color‑blind girls with normal fingers, 1/4 boys with normal vision and polydactyly B. 1/8 color‑blind girls with polydactyly, 1/8 boys with normal vision and normal fingers C. 1/4 girls with normal vision and polydactyly, 1/8 boys with normal vision and polydactyly D. 1/8 color‑blind girls with normal fingers, 1/4 boys with normal vision and polydactyly

B. 1/8 color‑blind girls with polydactyly, 1/8 boys with normal vision and normal fingers

How many genotypes are possible at a locus with five alleles? A. 5 B. 15 C. 10 D. 12

B. 15

The very rare Bombay blood phenotype in humans (first discovered in Bombay, India) results in type O blood because of the lack of both the A and B antigens in individuals who are of hh genotype. This genotype results in blood type O blood regardless of the genotype at the unlinked I locus. If two parents are both of IAIB Hh genotype (type AB blood), what is the probability that their first child will also have the AB blood type? A. 3/4 B. 3/8 C. 1/4 D. 1/162

B. 3/8

With a locus that contains five alleles, how many different homozygotes can there be? A. 32 B. 5 C. 10 D. 1

B. 5

A woman is phenotypically normal, but her father had the sex-linked recessive condition of red-green color blindness. If she marries a man with normal vision, what is the probability that their two children will both have normal vision? A. 4/9 B. 9/16 C. 1/16 D. 3/8 E. 3/4

B. 9/16

A homozygous beardless male goat is crossed with a homozygous bearded female goat. What are the phenotypes of the offspring? A. All offspring are beardless. B. All males are bearded; all females are beardless. C. All offspring are bearded. D. All males are beardless; all females are bearded.

B. All males are bearded; all females are beardless.

In Morgan's experiments involving white-eyed fruit flies (see panel (a) of Figure 4.12), which generation of fly crosses supported the hypothesis that white eyes were sex-linked? [Click on the image to view full size.] A. P generation B. F2 generation C. F1 generation D. F3 generation

B. F2 generation

Identical, or monozygotic, twins develop from a single egg fertilized by a single sperm. Monozygotic twins are genetically identical because they originate from a single zygote that split into two. Caroline Loat and her colleagues examined nine measures of social, behavioral, and cognitive ability in 1000 pairs of both male and female identical twins. Their study found that pairs of male twins tended to be more alike in their prosocial behavior, peer problems, and verbal ability scores than pairs of female twins. Which statement explains this observation? A. Interacting genes on the X and Y chromosomes in the brains of males results in identical twins with more similar behaviors. B. Females are mosaic for the expression of heterozygous X‑linked loci, because females undergo random X‑inactivation. C. Identical male twins express the same X‑linked alleles in their neural cells, because males undergo random X‑inactivation. D. Male twins have similar expression levels of X‑linked genes, because the Y chromosome silences expression of X chromosome genes.

B. Females are mosaic for the expression of heterozygous X‑linked loci, because females undergo random X‑inactivation.

How does the heterogametic sex differ from the homogametic sex? A. The heterogametic sex is male; the homogametic sex is female. B. Gametes of the heterogametic sex have different sex chromosomes; gametes of homogametic sex have the same sex chromosome. C. Gametes of the heterogametic sex all contain a Y chromosome. D. Gametes of the homogametic sex all contain an X chromosome

B. Gametes of the heterogametic sex have different sex chromosomes; gametes of homogametic sex have the same sex chromosome.

An X‑linked recessive gene causes red-green color blindness in humans. Suppose John and Cathy have normal color vision. After 10 years of marriage to John, Cathy has given birth to a color‑blind daughter and a color*#8209;blind son. John filed for divorce, claiming that he is not the father of at least one of the children. Which statement describes John's paternity claim? A. He could be the father of both children. B. He cannot be the father of Cathy's daughter. C. He cannot be the father of Cathy's son. D. He cannot be the father of either child.

B. He cannot be the father of Cathy's daughter.

What blood genotype is considered the universal receiver? A. IAi B. IAIB C. ii D. IBi

B. IAIB

Since the white‑eye trait is X‑linked recessive, what result would you expect when crossing a heterozygous red‑eyed female with a white‑eyed male? A. all red‑eyed progeny B. In each sex, half of the offspring will have red and half of the offspring will have white eyes. C. all red‑eyed females and all white‑eyed males D. all red‑eyed females with half of the males having white and half of the males having red eyes E. all red‑eyed males with half of the females having white and half of the females having red eyes

B. In each sex, half of the offspring will have red and half of the offspring will have white eyes.

What is the apparent purpose for X inactivation in humans and other mammals? A. It allows for the levels of expression of genes on the autosomes to be similar to the levels of genes on the X chromosome. B. It allows for the levels of expression of genes on the X chromosome to be similar in males and females. C. It suppresses the expression of genes on the Y chromosome in males. D. It reduces the amount of nondisjunction during meiosis in females. E. It enhances the level of pairing between the two X chromosomes during meiosis in females.

B. It allows for the levels of expression of genes on the X chromosome to be similar in males and females.

Which of the statements accurately describes both pleiotropy and polygenic inheritance? A. Polygenic inheritance refers to the cumulative effect of two or more genes on a single trait. Pleiotropy refers to a single gene that affects a single trait. B. Polygenic inheritance refers to the cumulative effect of two or more genes on a single trait. Pleiotropy refers to a single gene that affects multiple traits. C. Polygenic inheritance refers to a single gene that affects multiple traits. Pleiotropy refers to the cumulative effect of two or more genes on a single trait. D. Polygenic inheritance refers to the effect of a single gene on a single trait. Pleiotropy refers to a single gene that affects multiple traits.

B. Polygenic inheritance refers to the cumulative effect of two or more genes on a single trait. Pleiotropy refers to a single gene that affects multiple traits.

Why are most sex-linked characteristics X-linked? A. Both males and females have an X chromosome. B. The Y chromosome contains little genetic information. C. The pattern of inheritance for sex-linked characteristics is identical to that exhibited by genes located on autosomes. D. Most sex-linked disorders affect females.

B. The Y chromosome contains little genetic information.

In a cross between a white‑eyed female with two X chromosomes and one Y chromosome (XwXwY) with a red‑eyed male, the majority of the males have white eyes and the majority of the females have red eyes. However, rare red‑eyed males and white‑eyed females are present as well. What is the most likely chromosomal constitution of the red‑eyed males and white‑eyed females resulting from this cross? A. X+Y males and XwXw females B. X+Y males and XwXwY females C. X+XwY females and Xw males D. XwY males and XwX+ females E. X+XwY males and Xw females

B. X+Y males and XwXwY females

In the animation, the rare white‑eyed males generated from offspring of true‑breeding red‑eyed females and white‑eyed males were the result of a nondisjunction of X chromosomes in the female during meiosis I. Could white‑eyed males be generated in this cross by a nondisjunction of X chromatids in the female during meiosis II? Why or why not? A. No, if a nondisjunction of X chromatids occurred in meiosis II of a female, all progeny would have red eyes. B. Yes, because a nondisjunction of X chromatids in meiosis II of a female will generate two abnormal gametes, one with two X chromosomes and one with no X chromosome. C. No, if a nondisjunction of X chromatids occurred in meiosis II of a female, none of the resulting offspring would be viable. D. No, if a nondisjunction of X chromatids occurred in meiosis II of a female, only white-eyed females could be generated. E. Yes, because a nondisjunction of X chromatids in meiosis II of a female will result in both white-eyed males and white-eyed females.

B. Yes, because a nondisjunction of X chromatids in meiosis II of a female will generate two abnormal gametes, one with two X chromosomes and one with no X chromosome.

The Talmud, an ancient book of Jewish civil and religious laws, states that if a woman bears two sons who die of bleeding after circumcision (removal of the foreskin from the penis), any additional sons that she has should not be circumcised. The bleeding is most likely due to the X‑linked disorder hemophilia. Furthermore, the Talmud states that the sons of her sisters must not be circumcised, whereas the sons of her brothers should be. Is this religious law consistent with sound genetic principles? A. Yes, the woman is likely to be homozygous for the hemophilia allele since she has two sons with the disorder. Her sisters and her brothers are also likely to carry the mutant allele with both at risk for having affected sons. B. Yes, the woman is a carrier and half of her sons will be affected. Her sisters may also be carriers, but her brothers' sons will get their X chromosomes from their mothers, who are unlikely to be carriers of the allele. C. No, the woman is likely a carrier for the mutant allele, which would mean that half of her brothers are expected to carry this allele also and be at risk of having affected sons. Her sisters are likely to be homozygous normal and not at risk of having affected sons. D. No, the sons of both the woman's sisters and her brothers should be not circumcised. The allele for hemophilia is segregating in her family, and both her sisters and brothers may carry it and their sons may inherit it. E. No, the woman is a carrier for hemophilia, but her sisters are likely to be homozygous for the normal allele since it is unlikely that two sisters in the same family could both be both carriers and have affected sons.

B. Yes, the woman is a carrier and half of her sons will be affected. Her sisters may also be carriers, but her brothers' sons will get their X chromosomes from their mothers, who are unlikely to be carriers of the allele.

What is the phenomenon called where there is stronger or earlier expression of a genetic trait in succeeding generations? A. epistasis B. anticipation C. pleiotrophy D. variable expressivity E. reduced penetrance

B. anticipation

Characteristics that exhibit many overlapping phenotypes are called _____ characteristics. A. discontinuous B. continuous C. polygenic D. dominant pleiotropic

B. continuous

The results of which of these crosses allowed T. H. Morgan to conclude that the white‑eye trait in Drosophila was not autosomal recessive? A. crosses between true‑breeding red‑eyed females and red‑eyed males B. crosses between true-breeding white‑eyed females and red‑eyed males C. crosses between true‑breeding red‑eyed females and white‑eyed males D. None of the crosses would give a different result than what would be expected if the white‑eye trait was autosomal recessive. E. crosses between true‑breeding white‑eyed females and white‑eyed males

B. crosses between true-breeding white‑eyed females and red‑eyed males

The platypus is a sexually reproducing mammal in which the sexes are separate. Unlike other mammals, the male has five X and five Y chromosomes. The female has 10 X chromosomes. Select the term that describes the organization of the sexual organs in the platypus. A. intersex B. dioecious C. hermaphroditic D. monoecious

B. dioecious

DNA sequencing has revealed the presence of which feature on the human Y chromosome? A. a higher density of genes than on other chromosomes B. eight large palindromes (sequences that read the same on both strands of DNA) C. the presence of extra copies of the SRY gene on autosomes D. many genes involved with female fertility

B. eight large palindromes (sequences that read the same on both strands of DNA)

Anhidrotic ectodermal dysplasia is an X‑linked recessive disorder in humans characterized by small teeth, no sweat glands, and sparse body hair. The trait is usually seen in men, but women who are heterozygous carriers of the trait often have irregular patches of skin with few or no sweat glands (see the illustration). Irregular patches of skin lacking sweat glands in heterozygous female carriers of anhidrotic ectodermal dysplasia are caused by: A. the presence of one X chromosome in some cells and the presence of two X chromosomes in other cells. B. inactivation of the X chromosome with the wild‑type allele in the cells of the sweat gland lacking patches. C. inactivation of the X chromosome with the dysplasia allele in the cells of the sweat gland lacking patches. D. nondisjunction of the X chromosome during gametogenesis in the cells of the sweat gland lacking patches. E. the presence of two X chromosomes in some cells and the presence of an X and a Y chromosome in other cells. Why does the distribution of the patches of skin lacking sweat glands differ among the females depicted in the illustration, even between the identical twins? A. It is due to the random determination of the number of Barr bodies in each cell. B. It is due to random mutations in the two twins. C. It is due to random inactivation of different X chromosomes in different cells. D. It is due to placement of sweat glands being affected by environmental factors. E. It is due to the pattern of sex determination being different in the two twins.

B. inactivation of the X chromosome with the wild‑type allele in the cells of the sweat gland lacking patches. C. It is due to random inactivation of different X chromosomes in different cells.

The R locus determines flower color in a new plant species. Plants that are genotype RR have red flowers, and plants that are rr have white flowers. However, Rr plants have pink flowers. What type of inheritance does this demonstrate for flower color in these plants? A. complete dominance B. incomplete dominance C. codominance D. complementation E. lethal alleles

B. incomplete dominance

The allele H codes for LDL receptors that are needed to metabolize LDL. People who are HH have healthy levels of LDL, whereas those who are hh have dangerous levels of cholesterol that often lead to heart attacks and death during childhood. Interestingly, those who are Hh have high levels of cholesterol and are at risk of a heart attack as a young adult, but if their cholesterol is managed, they will have a long life. This is an example of _______. A. complete dominance B. incomplete dominance C. codominance D. recessive alleles E. None of the above

B. incomplete dominance

Males can inherit a gene for a sex-limited trait from their: A. mother's mother only. B. mother's or father's parents. C. father's mother only. D. mother's father only.

B. mother's or father's parents.

Martha has blood type AB. Where are the alleles for the A and B antigens located? A. on different, nonhomologous chromosomes B. on different homologous chromosomes C. on the same chromatid D. on sister chromatids

B. on different homologous chromosomes

In domestic chickens, some males display a plumage pattern called cock feathering. Other males and all females display a pattern called hen feathering. Cock feathering is an autosomal recessive trait that is exhibited in males only. What type of inheritance is exhibited by this trait? A. sex-linked B. sex-limited C. sex-influenced

B. sex-limited

Which statement below defines epistasis? A. when a heterozygote displays a phenotype intermediate between its two alleles B. when one locus affects or covers the outcome of another locus C. when a single gene affects several phenotypes D. a measurement of phenotype intensity

B. when one locus affects or covers the outcome of another locus

In mammals, each inactivated X chromosome forms a darkly staining body known as a _____ body. Barr Lyon Morgan Bridges

Barr

How many Barr bodies are present in an individual with Klinefelter syndrome (XXYY)?

Barr bodies= n-1 where n= number of X chromosomes answer: 2 Barr bodies

Studies of rare white-eyed males in Drosophila crosses by _____ demonstrated that the gene for white eyes is on the X chromosome and confirmed the chromosome theory of inheritance. Mendel Bridges Barr Klinefelter

Bridges

In domestic chickens, some males display a plumage pattern called cock feathering. Other males and all females display a pattern called hen feathering. Cock feathering is an autosomal recessive trait that is exhibited in males only. Two heterozygous birds are mated. What fraction of the male offspring is expected to exhibit cock feathering? A. 0 B. 1/8 C. 1/4 D. 1/2 E. 3/4

C. 1/4

In cats, curled ears result from an allele (Cu) that is dominant over an allele (cu) for normal ears. Black color results from an independently assorting allele (G) that is dominant over an allele for gray (g). A gray cat homozygous for curled ears is mated with a homozygous black cat with normal ears. All the F1 cats are black and have curled ears. An F1 cat mates with a stray cat that is gray and possesses normal ears. What phenotypes and proportions of progeny are expected from this cross? A. All black cats with curled ears B. 9/16 black cats, curled ears; 3/16 black cats, normal ears; 3/16 gray cats, curled ears; and 1/16 gray cats, normal ears C. 1/4 black cats, curled ears; 1/4 black cats, normal ears; 1/4 gray cats, curled ears and 1/4 gray cats, normal ears D. 9/16 gray cats, normal ears; 3/16 gray cats, curled ears; 3/16 black cats, normal ears; and 1/16 black cats, curled ears E. All gray cats; normal ears

C. 1/4 black cats, curled ears; 1/4 black cats, normal ears; 1/4 gray cats, curled ears and 1/4 gray cats, normal ears

The allele l in Drosophila is recessive, sex-linked, and lethal when homozygous or hemizygous (in males). There is no other phenotype associated with the mutant allele. If a female of genotype Ll is crossed with a normal male, what is the probability that the first two surviving flies will be males? A. 1/16 B. 1/2 C. 1/9 D. 1/32

C. 1/9

A maternal effect can cause the offspring phenotype ratio to depart from that of classic Mendelian inheritance. In a species of moth, the dominant allele N codes for brown eyes and recessive allele n codes for red eyes. If an Nn female with brown eyes mates with an Nn male, what is the eye color phenotypic ratio of their offspring? A. 3, orange eyes : 1, red eyes B. 3, brown eyes : 1, red eyes C. 4, brown eyes : 0, red eyes D. 0, brown eyes : 4, red eyes

C. 4, brown eyes : 0, red eyes

If there are five alleles at a locus, how many different kinds of homozygotes will there be? A. 1 B. 2 C. 5 D. 25 E. 3125

C. 5

A normal female Drosophila produces abnormal eggs that contain a complete diploid set of her chromosomes. She mates with a normal male Drosophila that produces normal sperm. What will the sex ratio of the progeny from this cross be? A. all females B. 25% male, 50% intersex, 25% female C. 50% female, 50% intersex D. 50% males, 50% females E. 50% female, 50% metafemale

C. 50% female, 50% intersex

A gene has three alleles. How many different genotypes are possible at this locus in a diploid organism? A. 5 B. 3 C. 6 D. 7 E. 4

C. 6

Epistasis often results in modified dihybrid phenotypic ratios. Assume that you obtain one such modified ratio, 9:7, with the gene pairs A and B involved. What would be a possible genotype for a phenotype that would be included with the 9 portion of the modified ratio? A. aa bb B. aa Bb C. Aa BB D. aa BB E. Aa bb

C. Aa BB

Suppose that the "fabulous" phenotype is controlled by two genes, A and B, as shown in the diagram below. Allele A produces enough enzyme 1 to convert "plain" to "smashing." Allele a produces no enzyme 1. Allele B produces enough enzyme 2 to convert "smashing" to "fabulous." Allele b produces no enzyme 2. The A and B genes are both autosomal and assort independently. What will be the phenotype(s) of the F1 offspring of a true-breeding "fabulous" father and a true-breeding "plain" mother (aa bb)? A. All "plain" B. All "smashing" C. All "fabulous" D. Plain" females and "fabulous" males E. "Fabulous" females and "smashing" males

C. All "fabulous"

How are mitochondrial genes typically inherited? A. An individual inherits their mitochondrial DNA from their father. B. An individual inherits one copy of mitochondrial DNA from each parent. C. An individual inherits their mitochondrial DNA from their mother. D. An individual randomly inherits their mitochondrial DNA from either parent.

C. An individual inherits their mitochondrial DNA from their mother.

In some reptilian species, temperature determines the sex of an embryo. The graph shows the effect of temperature on the percentage of male reptilian embryos, where Tp is the pivotal temperature. Select the statement that describes the influence of temperature on the percentage of male or female offspring as shown by the graph. A. Temperature only affects the percentage of male offspring, not female offspring. B. High temperatures do not affect sex. C. At high temperatures, nearly 100% of offspring will be male. D. At Tp, 100% of offspring will be male. E. At high temperatures, 0% are male offspring.

C. At high temperatures, nearly 100% of offspring will be male.

The very rare Bombay blood phenotype in humans (first discovered in Bombay, India) results in blood type O because of the lack of both the A and B antigens in individuals who are of hh genotype. This genotype results in blood type O regardless of the genotype at the unlinked I locus. The Bombay blood type demonstrates __________ because it suppresses the effect of the ABO alleles. A. Expressivity B. Penetrance C. Epistasis D. Pleiotropy E. Dominance

C. Epistasis

The difference between dominance and epistasis is that: A. Dominance masks genes at different loci. B. Epistasis masks genes at the same loci. C. Epistasis masks genes at different loci. D. All of the above. E. Dominance and epistasis are the same.

C. Epistasis masks genes at different loci.

In Drosophila, yellow body is due to an X-linked gene (Xy) that is recessive to the gene for gray body (X+). A yellow female is crossed with a gray male. The F1 are intercrossed to produce the F2. What proportion of the F2 progeny will be yellow? A. None of the F2 B. All of the males, none of the females C. Half of the males, half of the females D. None of the females, half of the males E. 1/4 of males, 1/4 of females

C. Half of the males, half of the females

X and Y chromosomes are not homologs, but in meiosis they do pair and segregate in XY organisms to create 50% haploid gametes with an X chromosome and 50% haploid gametes with a Y chromosome. How is pairing achieved? A. Since all other homologous chromosomes pair, the remaining two chromosomes pair by default. B. Pairing proteins are capable of binding to different genes on the X and Y chromosomes, which allows them to pair. C. Pseudoautosomal regions that are homologous exist at the tips of both the X and Y chromosomes, and they allow pairing. D. They don't actually pair. Random segregation generally ensures the X and Y chromosomes separate. E. None of the statements is correct.

C. Pseudoautosomal regions that are homologous exist at the tips of both the X and Y chromosomes, and they allow pairing.

What is a Barr body and how is it related to the Lyon hypothesis? A. The Barr body is a darkly staining structure found in the Y chromosome and was hypothesized by Mary Lyon to be the inactivated SRY gene. B. The Barr body is the parts of the X and Y chromosomes that are similar or homologous to each other, and Mary Lyon hypothesized that these regions pair together during meiosis. C. The Barr body is the darkly staining structure in the nuclei of female mammalian cells, and Mary Lyon hypothesized that it represented the inactive X chromosome. D. The Barr body is a long linear structure which is found in X chromosomes, and Mary Lyon hypothesized that it contained genes that are particularly active in females. E. The Barr body is located just outside the nucleus in female cells, and Mary Lyon hypothesized that it was the region where testosterone was synthesized.

C. The Barr body is the darkly staining structure in the nuclei of female mammalian cells, and Mary Lyon hypothesized that it represented the inactive X chromosome.

What does functional hemizygosity mean? A. The cells in an individual female are identical with respect to the expression of the genes on the X chromosome. B. In females that are heterozygous at an X-linked locus, all of the cells express one allele. C. The cells in an individual female are not identical with respect to the expression of the genes on the X chromosome. D. It refers to the degree of similarity of the alleles for a trait in an organism.

C. The cells in an individual female are not identical with respect to the expression of the genes on the X chromosome.

In the XX-XO system of sex determination, males have ______ for their sex chromosome(s). A. XX B. XY C. X D. YY E. Y

C. X

Female humans who are heterozygous for X-linked recessive gene disorders sometimes exhibit mild expression of the recessive trait. However, such mild expression of X-linked recessive traits in females who are heterozygous for X-linked alleles is not seen in Drosophila. What might cause this difference in the expression of X-linked genes in female humans and Drosophila? A. Doubling of the activity of both the dominant and recessive X-linked genes in Drosophila results in the expression of only the dominant traits, while in human females both dominant and recessive traits are expressed. B. Recessive X-linked traits in human females are occasionally expressed due to sex-influenced factors that are absent in Drosophila females as result of the dosage compensation by doubling of the activity of genes on the X chromosome. C. X inactivation in human females results in females who are mosaics for X-linked heterozygous loci, resulting in some cells expressing the dominant allele and some cells expressing the recessive allele. X inactivation does not occur in Drosophila; thus, all heterozygous cells express both the dominant and recessive alleles. D. X inactivation in Drosophila females heterozygous for X-linked traits results in females that express only the X-linked dominant allele, while X inactivation in human females results in females that are mosaics for X-linked heterozygous loci, resulting in some cells expressing the dominant allele and some cells expressing the recessive allele. E. Incomplete penetrance of most dominant X-linked alleles within human females results in females whose recessive allele is expressed. X-linked incomplete penetrance of dominant alleles is not seen in Drosophila due to dosage compensation.

C. X inactivation in human females results in females who are mosaics for X-linked heterozygous loci, resulting in some cells expressing the dominant allele and some cells expressing the recessive allele. X inactivation does not occur in Drosophila; thus, all heterozygous cells express both the dominant and recessive alleles.

In what scenario would a human typically display male features? A. a defect in the SRY gene in an XX individual B. a defect in the receptor that binds testosterone in an XY individual C. a fragment of the Y chromosome containing the SRY gene attached to one X chromosome in an XX individual D. individuals with androgen-insensitivity syndrome (XX genotype)

C. a fragment of the Y chromosome containing the SRY gene attached to one X chromosome in an XX individual

What is the definition of a hypostatic gene? A. the masking of the expression of one gene by another gene at a different locus B. a single copy of an allele that inhibits the expression of an allele at a different locus C. a gene that is suppressed by the effect of another gene at a different locus D. a gene that suppress the effect of other genes at a different locus

C. a gene that is suppressed by the effect of another gene at a different locus

Hypospadias, a birth defect in male humans in which the urethra opens on the shaft instead of at the tip of the penis, results from an autosomal dominant gene in some families. Females who carry the gene show no effects. This birth defect is an example of: A. an X-linked trait. B. a Y-linked trait. C. a sex-limited trait. D. a sex-influenced trait. E. genetic maternal effect.

C. a sex-limited trait.

In XX-XO sex determination, males have which sex chromosomes? A. two X chromosomes B. three X chromosomes C. a single X chromosome D. one X chromosome and one Y chromosome

C. a single X chromosome

In azaleas, the leaves and stems may be green, white, or variegated (mixture of green and white splotches). Female flowers from white plants were crossed using pollen from white plants, and the offspring were all white. If these same female flowers from white plants were crossed using pollen from green plants, what would be the phenotypic ratio in the offspring? A. 3 green : 1 white B. all variegated C. all white D. half green and half white

C. all white

Huntington's disease (HD) is an autosomal dominant condition that is typically not expressed until middle age. However, children receiving the mutant allele from their father often express the condition earlier in life. What term describes this situation? A. variable expressivity B. genomic imprinting C. anticipation D. reduced penetrance

C. anticipation

Which statement does not explain why tortoiseshell cats are usually female? A. because of X inactivation in females B. because the gene for the presence of orange color is on the X chromosome C. because cats have three copies of the X chromosome D. because males only have one copy of the X-linked locus for orange color

C. because cats have three copies of the X chromosome

Which of these is an example of dominant epistasis? A. seed shape and seed color in peas B. coat color in Labrador retrievers C. color in squash D. albinism in snails

C. color in squash

Species in which individuals have only male or only female reproductive structures are called: A. hermaphrodites B. diploids C. dioecious D. homogametic E. monoecious

C. dioecious

Flower color is determined by two gene pairs in a species of plant. Two red flowered plants are crossed and produce offspring in the phenotypic ratio of 12 red : 3 purple : 1 white. What kind of gene interaction is this? A. recessive epistasis B. dominant and recessive epistasis C. dominant epistasis D. duplicate recessive epistasis

C. dominant epistasis

In Caenorhabditis elegans, the level of expression of genes on both X chromosomes of females is reduced by half. What phenomenon is this an example of? A. genomic imprinting B. extranuclear inheritance C. dosage compensation D. paternal mitochondrial leakage

C. dosage compensation

Fruit flies with XX sex chromosomes and three haploid sets of autosomes (AAA) have which sexual phenotype? A. male B. metamale C. intersex D. female

C. intersex

Sponges can produce sperm and eggs in a single organism. Which term describes the sex system of the organism? A. XY sex determination B. ZW sex determination C. monoecious D. XO sex determination E. dioecious

C. monoecious

The SRY gene in mammals is normally carried on which of these chromosomes? A. the Z chromosome B. both the X and the Y chromosomes C. the Y chromosome D. the X chromosome

C. the Y chromosome

A correct explanation for why tortoiseshell cats are almost always female and have a patchy distribution of orange and black fur might be that: A. orange coat color is not found in male cats. B. black coat color is not found in male cats. C. tortoiseshell cats have two different X-linked alleles, X+ (black) and Xo (orange). Random X-inactivation during early development results in patches of orange and black. Males with only a single X chromosome will be either orange or black. D. tortoiseshell cats have two different X-linked alleles, X+ (black) and Xo (orange). Patchy coat coloration in females results from equal expression of both the X+ and Xo, with neither being completely dominant over the other. Males with only a single X chromosome will be either orange or black. E. None of the above choices are adequate explanations for why tortoiseshell cats are almost always females.

C. tortoiseshell cats have two different X-linked alleles, X+ (black) and Xo (orange). Random X-inactivation during early development results in patches of orange and black. Males with only a single X chromosome will be either orange or black.

The dominance pattern of a gene can be determined from the phenotypes of the parents and offspring. In the examples below, assume that each parent is homozygous for the specific allele and that the progeny are heterozygous. Classify each example as either complete dominance, incomplete dominance, or codominance. - Complete dominance? - Incomplete dominance? - Codominance? Answer bank: - A black sheep and a white sheep produce a gray lamb. - A mother with straight hair and a father with curly hair have a son with wavy hair. - A mother with type A blood and a father with type B blood have a daughter with type AB blood. - A pea plant with all purple flowers and a pea plant with all white flowers produce a pea plant with all purple flowers. - A white cow and a red bull have a calf that is white with red spots (roan colored).

Complete dominance: - A pea plant with all purple flowers and a pea plant with all white flowers produce a pea plant with all purple flowers. Incomplete dominance: - A black sheep and a white sheep produce a gray lamb. - A mother with straight hair and a father with curly hair have a son with wavy hair. Codominance: - A mother with type A blood and a father with type B blood have a daughter with type AB blood. - A white cow and a red bull have a calf that is white with red spots (roan colored).

A woman has blood type A MM. Her child has blood type AB MN. The blood types of the potential fathers are given in the table. Chart layout: [Name; Blood type] [George; O NN] [Tom; AB MN] [Bill; B MN] [Claude; A NN] [Henry; AB MM] Based on their blood types, which of these men could be the father of this child? - Could be the father? - Not the father? Answer bank: - George - Bill - Henry - Tom - Claude

Could be the father: - Tom - Bill Not the father: - George - Claude - Henry

A man, Joe, has classic hemophilia, an X‑linked recessive disease. Classify each person depending on whether or not Joe could have inherited the hemophilia gene from him or her. - Could have inherited? - Could not have inherited? Answer bank: - paternal grandfather - paternal grandmother - maternal grandfather - maternal grandmother

Could have inherited: - maternal grandfather - maternal grandmother Could not have inherited: - paternal grandfather - paternal grandmother

Dominant epistasis is seen in the interaction of two loci that determine fruit color in summer squash. When a homozygous plant that produces white squash is crossed with a homozygous plant that produces green squash and the F1 plants are crossed with each other, what is the resultant genotype ratio of the F2 generation? A. 9:7 B. 9:3:4 C. 9:3:3:1 D. 12:3:1

D. 12:3:1

If nondisjunction of X chromosomes occurs at a rate of 2% in a certain population of Drosophila, approximately how many white‑eyed males would you expect to see among 1000 offspring resulting from breeding a true‑breeding red‑eyed female with a white‑eyed male? A. 10 B. 2 C. This is impossible to calculate with the given information. D. 20 E. 5

D. 20

A mother with blood type B has a child with blood type O. Give all possible blood types for the father of this child. A. O B. B, AB C. A, AB D. A, B, O E. A, B, AB, O

D. A, B, O

Because the white‑eye trait is X‑linked recessive, what result would you expect when crossing a heterozygous red‑eyed female with a red-eyed male? A. All females will be red‑eyed and all males will be white‑eyed. B. All offspring will be red-eyed. C. In both sexes, half of the offspring will have red eyes and half of the offspring will have white eyes. D. All females will have red eyes whereas half of the males will have red eyes and half will have white eyes. E. All males will be red-eyed whereas half of the females will have red eyes and half will have white eyes.

D. All females will have red eyes whereas half of the males will have red eyes and half will have white eyes.

What characteristic is exhibited by a Y-linked trait? A. Y-linked traits appear only in male offspring of men with the trait. B. Y-linked traits are only passed from father to son. C. Y-linked traits can be passed from mother to son. D. Both A and B are characteristics of Y-linked traits. E. Both A and C are characteristics of Y-linked traits.

D. Both A and B are characteristics of Y-linked traits.

A woman has blood type AM. She has a child with blood-type AB MN. Which of the following blood types could not be that of the child's father and why? George: O; N Tom: AB; MN Bill: B;MN Claude: A; N Henry: AB; M A. Only George and Claude are excluded because they do not have a B allele. B. Only Henry is excluded because he lacks an N allele. C. Only George is excluded because the child cannot have an O allele. D. George, Claude, and Henry are excluded because they lack either a B allele or an N allele. E. Any of these men could have been the father.

D. George, Claude, and Henry are excluded because they lack either a B allele or an N allele.

How does sex determination in Drosophila differ from sex determination in humans? A. In Drosophila, sex determination is environmental with the sexual phenotype being determined by the temperature during embryonic development while in humans X and Y sex chromosomes determine sex. B. Drosophila uses a XX-XO system of sex determination with males having one X chromosome and females having two X chromosomes while humans use a XX-XY system of sex determination. C. In Drosophila, the presence of a Y chromosome determines the male sex and flies with one X chromosome without a Y are female while in humans the presence of two X chromosomes produces the female sex while a single X chromosome without a Y chromosome results in the male sex. D. In Drosophila, sex is determined by the X:A ratio (the number of X chromosomes divided by the number of haploid sets of autosomal chromosomes), while in humans the presence of a Y chromosome produces the male sex and the absence of a Y chromosome results in the female sex. E. In Drosophila, an individual with two sets of autosomes and XXY chromosomes would be a male, while in humans such an individual would be a female.

D. In Drosophila, sex is determined by the X:A ratio (the number of X chromosomes divided by the number of haploid sets of autosomal chromosomes), while in humans the presence of a Y chromosome produces the male sex and the absence of a Y chromosome results in the female sex.

Who was the scientist who first discovered darkly staining bodies in nuclei of cells from female cats which were later shown to be the inactive X chromosomes? A. Calvin Bridges B. Thomas Hunt Morgan C. Mary Lyon D. Murray Barr

D. Murray Barr

Leber hereditary optic neuropathy (LHON) is a human disease that exhibits cytoplasmic inheritance. It is characterized by rapid loss of vision in both eyes, resulting from the death of cells in the optic nerve. A teenager loses vision in both eyes and is later diagnosed with LHON. How did this individual MOST likely inherit the mutant DNA responsible for this condition? A. a nuclear gene from the father B. a nuclear gene from the mother C. a mitochondrial gene from the father D. a mitochondrial gene from the mother E. Any of these answers is possible.

D. a mitochondrial gene from the mother

Two loci control body color in house flies. In a cross between a black fly and a gray fly, you obtain a 15:1 ratio, respectively. What kind of gene interaction is this? A. dominant and recessive B. epistasis C. duplicate recessive epistasis D. duplicate dominant epistasis E. recessive epistasis

D. duplicate dominant epistasis

The human genetic disease phenylketonuria (PKU) is caused by an autosomal recessive mutant allele. If left untreated, it leads to a high level of cognitive impairment due to toxic levels of phenylalanine. However, placing a child on a low-phenylalanine diet prevents the development of the intellectual disability. This situation is an example of which of these? A. a multifactorial characteristic B. a trait showing genetic maternal effects C. a temperature-sensitive allele that is active only at higher temperatures D. environmental effect on genotype expression

D. environmental effect on genotype expression

What type of gene action occurs when one gene masks the effect of another gene at a different locus? A. genomic imprinting B. anticipation C. complementation D. epistasis E. epigenetics

D. epistasis

What happens in the absence of testosterone and anti-Mullerian hormone? A. inactivation of an X chromosome B. degeneration of the female reproductive ducts C. male features develop D. female features develop

D. female features develop

A fruitfly is found that has three sex chromosomes (XXY) and three of each autosome. The sex of this fruitfly is _________ because ___________. A. female; it has a 1:1 ratio of sex chromosomes to autosomes. B. male; it has a Y chromosome. C. female; it has 2 X chromosomes. D. intersex; it has an X:A ratio between 0.5 and 1. E. impossible to determine; sex is influenced by environment in insects.

D. intersex; it has an X:A ratio between 0.5 and 1.

Leber hereditary optic neuropathy (LHON) is a human disease that results from a mutation in the mitochondrial DNA. It is characterized by a rapid loss of vision in both eyes, resulting from the death of cells in the optic nerve. Assume that an affected man marries an unaffected woman. What proportion of their children is going to be affected with LHON? A. one-half B. one-fourth C. all D. none

D. none

In a __________, environmental factors alone can produce a phenotype that is the same as the phenotype produced by a genotype. A. compound heterozygote B. genetic maternal effect C. polygenic characteristic D. phenocopy E. pleiotropy

D. phenocopy

In domestic chickens, some males display a plumage pattern called cock feathering. Other males and all females display a pattern called hen feathering. Cock feathering is an autosomal recessive trait that is exhibited in males only. Two heterozygous birds are mated. What fraction of the offspring is expected to exhibit hen feathering? A. one-fourth B. all C. three-fourths D. seven-eighths

D. seven-eighths

A recessive mutant allele of an autosome gene in a species of mouse results in a shorted tail in males when homozygous. However, when homozygous in females, this genotype has no effect, and the mice have normal tails. What is this genetic phenomenon called? A. genomic imprinting B. cytoplasmic inheritance C. genetic maternal effect D. sex‑limited characteristic E. sex‑influenced characteristic

D. sex‑limited characteristic

Multifactorial characteristics result from which of the statements? A. the activity of a gene locus where there are multiple alleles B. temperature‑sensitive alleles that are showing epigenetic expression C. complementation between mutant alleles in different genes D. the activity of polygenes influenced by environmental factors E. the activity of compound heterozygotes

D. the activity of polygenes influenced by environmental factors

With genetic maternal effect, the phenotype of an individual is determined by which of the statements? A. the sex of the individual with only one sex able to express the genotype B. cytoplasmic genes usually located in the mitochondria C. a combination of environmental factors and the genotype of the individual D. the nuclear genotype of the maternal parent E. the sex of the parent who transmits the gene to the individual

D. the nuclear genotype of the maternal parent

X-linked Characteristics:

Dosage compensation: the amount of protein produced by X-linked genes and randomly inactivated in two sexes

Crossing two yellow mice results in 2/3 yellow offspring and 1/3 nonyellow offspring. What percentage of offspring would you expect to be nonyellow if you crossed two nonyellow mice? Remember that a 2:1 ratio means that one genotype is lethal. A. 25% B. 33% C. 66% D. 75% E. 100%

E. 100%

In chickens, the dominant allele Cr produces the creeper phenotype (having extremely short legs). However, the creeper allele is lethal in the homozygous condition. The homozygous recessive genotype results in a normal individual. If two creepers are mated, what will be the phenotypic ratio among the living offspring? A. 1 normal : 1 creeper B. 3 creepers : 1 normal C. 3 normal : 1 creeper D. 2 normal : 1 creeper E. 2 creepers : 1 normal

E. 2 creepers : 1 normal

In humans, occasionally a baby is found that has the XY chromosomal karyotype but is phenotypically female. Which of the following statements might be a CORRECT explanation for at least some of these unusual cases? A. The ratio of number of X chromosomes to number of sets of chromosomes is incorrect. B. A small piece of the X chromosome is duplicated but too small to be detected. C. A small piece of the X chromosome is missing but is too small to be detected. D. An extra piece of autosomal chromosome 15 is probably present in the genome but is too small to be detected. E. A mutation has occurred in the SRY gene, making it inactive.

E. A mutation has occurred in the SRY gene, making it inactive.

In this chapter, we considered Joan Barry's paternity suit against Charlie Chaplin and how, on the basis of blood types, Chaplin could not have been the father of her child. Barry had blood type A, her child had blood type B, and Chaplin had blood type O. What blood types are possible for the father of Barry's child? A. A only B. B only C. AB only D. A or B E. AB or B

E. AB or b

If the white‑eye trait were Y‑linked dominant, rather than X‑linked recessive, what result would you expect when crossing a red‑eyed female with a white‑eyed male? A. All offspring will be red‑eyed. B. All female offspring will be white‑eyed, whereas half of the male offspring will have white eyes and half of the male offspring will have red eyes. C. All male offspring will be red‑eyed male, whereas half of the female offspring will have white eyes and half of the female offspring will have red eyes. D. For each sex, half of the offspring will have red eyes and half of the offspring will have white eyes. E. All female offspring will be red‑eyed and all male offspring will be white‑eyed.

E. All female offspring will be red‑eyed and all male offspring will be white‑eyed.

If the white-eye trait were X‑linked dominant, rather than X‑linked recessive, what result would you expect when crossing a heterozygous white‑eyed female with a white‑eyed male? A. All offspring will be red‑eyed. B. All male offspring will be red‑eyed, whereas half of the female offspring will have white eyes and half of the female offspring will have red eyes. C. For each sex, half of the offspring will have red eyes and half of the offspring will have white eyes. D. All female offspring will be red‑eyed, whereas half of the male offspring will have white eyes and half of the male offspring will have red eyes. E. All female offspring will be white-eyed, where as half of the male offspring will have white eyes and half of the male offspring will have red eyes.

E. All female offspring will be white-eyed, where as half of the male offspring will have white eyes and half of the male offspring will have red eyes.

Which of the following codes for a phenotypic human male? A. XY with the SRY gene located on an autosomal chromosome? B. XX with a copy of SRY gene on an autosomal chromosome? C. XO with a copy of SRY gene on an autosomal chromosome? D. XXYY with one copy of the SRY gene deleted? E. All of the above sex compliments results in a person with a sex phenotype of male.

E. All of the above sex compliments results in a person with a sex phenotype of male.

What characteristic is exhibited by an X-linked trait? A. Only females have the trait. B. Males inherit X-linked traits from their paternal parent only. C. Females inherit X-linked traits from their maternal parent only. D. Males can inherit a gene for an X-linked trait from either parent but normally inherit it from their fathers. E. Females can inherit a gene for an X-linked trait from either parent.

E. Females can inherit a gene for an X-linked trait from either parent.

If the white‑eye trait were X‑linked dominant rather than X‑linked recessive, what result would you expect when crossing a heterozygous white‑eyed female with a red‑eyed male? A. All female offspring will be red‑eyed, whereas half of the male offspring will have white eyes and half of the male offspring will have red eyes. B. All female offspring will be red‑eyed and all male offspring will be white‑eyed. C. All offspring will be red‑eyed. D. All male offspring will be red‑eyed, whereas half of the female offspring will have white eyes and half of the female offspring will have red eyes. E. For each sex, half of the offspring will have red eyes and half of the offspring will have white eyes.

E. For each sex, half of the offspring will have red eyes and half of the offspring will have white eyes.

In the yawncat (a rare hypothetical animal), the dominant allele R causes solid tail color, and the recessive allele r results in white spots on a colored background. The black coat color allele B is dominant to the brown allele b, but these genes can only be expressed if the animal has an mm genotype at a third gene locus. Animals that are M_ are yellow regardless of which allele from the B locus is present. A mating between a solid yellow-tailed male yawncat and a solid brown-tailed female yawncat produces 16 offspring with the following tail phenotypes: six solid yellow, two spotted yellow, three solid black, one spotted black, three solid brown, and one spotted brown. What is the most likely genotype of the male parent? A. MM BB RR B. MM Bb RR C. Mm Bb RR D. Mm BB Rr E. Mm Bb Rr

E. Mm Bb Rr

In which of the following individuals would you expect to find two Barr bodies in their somatic cells? A. XX B. XO C. XXY D. XXYY E. XXX

E. XXX

A man and a woman are both deaf due to being homozygous for a recessive autosomal mutant allele. However, they are homozygous recessive at different gene loci. If all their children have normal hearing, which event has occurred within each child? A. incomplete dominance B. epistasis C. gene interaction D. codominance E. complementation

E. complementation

In humans, mitochondrial genetic disorders are inherited from only the mother. The severity of such diseases can vary greatly, even within a single family. What form of inheritance does this represent? A. sex‑limited inheritance B. sex‑influenced inheritance C. genomic imprinting D. genetic maternal effect E. cytoplasmic inheritance

E. cytoplasmic inheritance

A gene whose expression is affected by the sex of the transmitting parent demonstrates which of the events? A. epigenetics B. anticipation C. pleiotropy D. genetic maternal effect E. genomic imprinting

E. genomic imprinting

In which of the following organisms is gender/sex determined by the temperature during embryonic development? A. humans B. mice C. fruit flies D. many snakes and birds E. many turtles and alligators

E. many turtles and alligators

The SRY gene is located on the Y chromosome. This single gene encodes a protein called a transcription factor that binds to DNA and stimulates the transcription of other genes that lead to the development of male sex characteristics, including physical, biochemical, and behavioral phenotypes. What concept in genetics BEST describes this example? A. dominance B. discontinuous characteristic C. polygenic characteristic D. phenocopy E. pleiotropy

E. pleiotropy

The himalayan allele in rabbits produces dark fur at the extremities of the body-- on the nose, ears, and feet. The dark pigment develops, however, only when a rabbit is reared at a temperature of 25^C or lower; if a Himalayan rabbit is reared at 30^C, no dark patches develop. What does this exemplify? A. dominance B. discontinuous characteristic C. genetic imprinting D. phenocopy E. temperature-sensitive allele

E. temperature-sensitive allele

What is penetrance? A. a situation where an allele is only expressed in one sex and does not produce the same phenotype in the other sex B. a situation where the stronger or earlier expression of a genetic trait occurs in succeeding generations C. the degree to which a trait is expressed D. a situation where a trait is only expressed at a particular temperature E. the percentage of individuals having a particular genotype who express the expected phenotype

E. the percentage of individuals having a particular genotype who express the expected phenotype

[True or False?]: Cytoplasmically inherited traits are always passed from father to daughter.

FALSE

[True or False?]: Male and female progeny often displaying different phenotypes is a characteristic exhibited by a cytoplasmically inherited trait.

FALSE

If an XY individual has a defective androgen receptor what external characteristics would the person exhibit?

Female

Miniature wings, Xm, in Drosophila melanogaster result from an X‑linked allele that is recessive to the allele for long wings, X+. Match the genotypes for each parent in the crosses. - Male Parent Genotype? - Female parent genotype?

Male Parent Genotype: - X+Y, XmY, XmY, X+Y, X+Y Female parent genotype: - X+Xm, X+X+, X+Xm, XmXm, X+X+

Suppose a graduate student is studying a loss‑of‑function mutation in the mouse gene zigzag. Whereas wild‑type mice have straight tails, zigzag mutant mice have tails with two sharp kinks, so that the tail looks like the letter Z. To determine how the zigzag phenotype is inherited, he performs the crosses listed in the first column of the table, using parents from true breeding lines. Three possible sets of results from these crosses are shown. Determine the mode of inheritance of the zigzag gene that would yield each result set. Chart layout: [Cross; Set A; Set B; Set C] [Cross 1: zigzag female X wild-type male; (SET A) zigzag males wild-type females; (SET B) all zigzag; (SET C) all wild-type] [Cross 2: Intercross F1 from cross 1; (SET A) 1/2 zigzag, 1/2 wild-type equally distributed among males and females; (SET B) all zigzag; (SET C) 1/2 zigzag, 1/2 wild-type equally distributed among males and females] [Cross 3: wild-type female X zigzag male; (SET A) all wild-type; (SET B) all wild-type; (SET C) all zigzag] [Cross 4: Intercross F1 from cross 3; (SET A) 1/4 zigzag males, 1/4 wild-type males, 1/2 wild-type females; (SET B) all wild-type; (SET C) 1/2 zigzag, 1/2 wild-type equally distributed among males and females] - Mode of inheritance? Answer bank: - genomic imprinting (maternal allele is inactive) - maternal effect inheritance - X-linked recessive inheritance - genomic imprinting (paternal allele is inactive) - cytoplasmic inheritance

Mode of inheritance: [listed from: left to right] - *X-linked recessive inheritance* - *cytoplasmic inheritance* - *genomic imprinting (maternal allele is inactive)*

An antigen found on red blood cells, Xg, is encoded by an X‑linked allele (𝑋𝑎) that is dominant over an allele for the absence of the antigen (𝑋−). Scientists studied the inheritance of these X‑linked alleles in children with chromosome abnormalities to determine where nondisjunction of the sex chromosomes occurred. Match the genotypes of the parents and offspring to the parent and cell division stage in which the nondisjunction must have occurred. Shown as parent×parent=child. - Nondisjunction in the mother during meiosis I or II? - Nondisjunction in the father during meiosis I or II? - Nondisjunction in the father during meiosis I? - Nondisjunction in the mother during meiosis II?

Nondisjunction in the mother during meiosis I or II: - X^aY x X^-X^- = X^a Nondisjunction in the father during meiosis I or II: - X^aY x X^aX^- = X^- Nondisjunction in the father during meiosis I: - X^aY x X^-X^- = X^aX^-Y Nondisjunction in the mother during meiosis II: - X^aY x X^aX^- = X^-X^-Y

Coat color in cats is determined by genes at several different loci. At one locus on the X chromosome, one allele (X^+) encodes black fur and another allele (X^o) encodes orange fur. Females can be black (X^+X^+), orange (X^oX^o), or a mixture of orange and black called tortoiseshell (X^+X^o). Males are either black (X^+Y) or orange (X^oY). Bill has a female tortoiseshell cat named Patches. One night, Patches escapes from Bill's house, spends the night out, and mates with a stray male. Patches later gives birth to the following kittens: one orange male, one black male, two tortoiseshell females, and one orange female. What are the genotypes of Patches, the stray male, and the kittens? - Patches? - Stray Male? - Orange Male Kitten? - Black Male Kitten? - Tortoiseshell Female Kitten? - Orange Female Kitten? Answer bank: - X^oY - X^+X^+ - X^+Y - X^oX^o - X^+X^o

Patches: - X^+X^o Stray Male: - X^oY Orange Male Kitten: - X^oY Black Male Kitten: - X^+Y Tortoiseshell Female Kitten: - X^+X^o Orange Female Kitten: - X^oX^o

Classify whether each gene regularly exists in a hemizygous state. - Regularly hemizygous? - Not regularly hemizygous? Answer bank: - a color-blindness gene on the X chromosome in a normal human male - a transgenic gene inserted into only one chromosome of a mouse - a gene on chromosome 21 in an individual with Down syndrome - a color-blindness gene on an X chromosome in a normal human female

Regularly hemizygous: - a color-blindness gene on the X chromosome in a normal human male - a transgenic gene inserted into only one chromosome of a mouse Not regularly hemizygous: - a gene on chromosome 21 in an individual with Down syndrome - a color-blindness gene on an X chromosome in a normal human female

Suppose in the Magudon people of Glaxoon that red, writhing tentacles are considered remarkably attractive in a mate. Most Magudons have red, floppy tentacles. Red, writhing tentacles have been found to run in families, but the pattern of inheritance has confounded Glaxooni geneticists. The Magudon inheritance of traits is remarkably similar to that of humans, and many traits exhibit classic Mendelian inheritance. Oddly, red, writhing tentacles do not follow a Mendelian inheritance pattern. Earth's first interplanetary geneticists have been asked to bring a new perspective to this intriguing problem. After studying the appearance of the trait in several families, the Earth geneticists determine that the red, writhing tentacles trait is autosomal recessive and exhibits a pattern of genetic imprinting. The maternal copy of the gene is imprinted. A family tree was created for one Magudon family. Both of the parents in generation I are heterozygous for tentacle movement. Individuals 5 and 10 are homozygous for the dominant trait, red, floppy tentacles. Statement 1: Individual 1 is heterozygous for tentacles and displays the recessive trait, red, writhing tentacles. The recessive allele was inherited from Individual 1's _________, and the _________ maternal copy of the gene has been silenced. Statement 2: The children of individuals 1 and 2 do not show the phenotypic frequencies usually found in an autosomal, heterozygous cross. The frequency of the children from individuals 1 and 2 with the recessive phenotype, red, writhing tentacles, is one-half. The usual phenotypic frequency of the recessive phenotype from this type of cross is and received their recessive allele from their _________. Statement 3: Some of the children of individuals 5 and 6 have red, writhing tentacles. Individual 5 has been identified as homozygous for red, floppy tentacles. In order for any of the children to display red, writhing tentacles, individual 6 must be _________ and must have received the recessive allele from his _________. Answer bank: - dominant - recessive - father - mother - phenotype - genotype - homozygous - heterozygous

Statement 1: - father; - dominant Statement 2: - heterozygous; - father Statement 3: - heterozygous; - mother

[True or False?]: Progeny from the same cross often displaying phenotypic variability is a characteristic exhibited by a cytoplasmically inherited trait.

TRUE

Y chromosome:

The male-determining gene is located on the Y chromosome. A single Y, even in the presence of several X's, still produces a male phenotype. The absence of Y results in a female phenotype.

Turner syndrome:

XO; 1/3000 female births

In Drosophila melanogaster, white eye is an X‑linked recessive trait, and red eye is an X‑linked dominant trait. Assume that the flies have a diploid set of autosomes (2A). Allele Xw codes for white eye, and allele X codes for red eye. Match each of the genotypes with the expected sex and eye color. Sex/Eye color? - XX? - X[^W]X? - X [^W]Y? - XO? - X[^W]XY? - X[^W]X[^W]X? Answer bank: - female - male - metafemale - white - red

XX: - female - red X[^W]X: - female - red X [^W]Y: - male - white XO: - male - red X[^W]XY: - female - red X[^W]X[^W]X: - metafemale - red

Poly-X females:

XXX (normal intelliegence, more than 3 X's increases intellectual disability)

Klinefelter syndrome:

XXY, or XXXY, or XXXXY, or XXYY; 1/1000 male births

Male‑limited precocious puberty results from a rare, sex‑limited autosomal allele (P) that is dominant over the allele for normal puberty (p) and is expressed only in males. Bill undergoes precocious puberty, but his brother Jack and his sister Beth underwent puberty at the usual time, between the ages of 10 and 14. Although Bill's mother and father underwent normal puberty, his two maternal uncles (his mother's brothers) underwent precocious puberty. All of Bill's grandparents underwent normal puberty. Match the most likely genotypes for each relative in his family. If there is an equal likelihood of multiple genotypes for a relative, place each possible genotype. Chart layout: [Relative; Genotype] [Bill; ______ ?] [Bill's brother Jack; _____?] [Bill's sister Beth; ______?] [Bill's father; ______?] [Bill's mother; ______?] [Bill's uncle; ______?] [Bill's grandfather; ______?] [Bill's grandmother; ______?] Answer bank: - Pp - pp - PP

[Bill; *Pp*] [Bill's brother Jack; *pp*] [Bill's sister Beth; *Pp; pp*] [Bill's father; *pp*] [Bill's mother; *Pp*] [Bill's uncle; *Pp*] [Bill's grandfather; *pp*] [Bill's grandmother; *Pp*]

Suppose the ear length of two populations of jerboas is controlled by one gene. To determine the mode of inheritance, a homozygous short‑eared female is crossed with a homozygous long‑eared male. Then, siblings from the F1 are crossed, and the number of short‑ and long‑eared animals are counted to determine the phenotype ratio. Match the phenotypic ratio anticipated in the offspring from a cross of two heterozygous individuals to the appropriate mode of inheritance for the short‑ear allele. Chart layout: [Mode of inheritance; Autosomal dominant; Autosomal recessive; Sex-linked dominant; Sex-linked recessive] [F1 phenotype; All short-eared; All long-eared; All short-eared; Long-eared females, short-eared males] [Cross of heterozygotes; *______, ______, _______, _______ ]*? Answer bank: - male 1 long : 1 short; females all short - 1 long : 3 short - 1 long : 1 short - 3 long : 1 short

[Cross of heterozygotes; *1 long : 3 short; 3 long : 1 short; males 1 long : 1 short; females all short; 1 long : 1 short]*

Match the sexual phenotype of fruit flies that have the listed chromosomes. Chart layout: [Sex chromosomes; Autosomal chromosomes; Sexual phenotype] [XX; all normal; *________* ] [XY; all normal; *________* ] [XO; all normal; *_______* ] [XXY; all normal; *_______* ] [XYY; all normal; *_______* ] [XXYY; all normal; *_______* ] [XXX; all normal; *________* ] [XX; 4 haploid sets; *_______ *] [XXX; 4 haploid sets; *________* ] [XXX; 3 haploid sets; *________* ] [X; 3 haploid sets; *_______* ] [XY; 3 haploid sets; *______* ] [XX; 3 haploid sets; *______* ] Answer bank: - metafemale - sterile male - intersex - male - female - sterile metamale - metamale

[XX; all normal; *female*] [XY; all normal; *male*] [XO; all normal; *sterile male*] [XXY; all normal; *female*] [XYY; all normal; *male*] [XXYY; all normal; *female*] [XXX; all normal; *metafemale*] [XX; 4 haploid sets; *sterile male*] [XXX; 4 haploid sets; *intersex*] [XXX; 3 haploid sets; *female*] [X; 3 haploid sets; *sterile metamale*] [XY; 3 haploid sets; *metamale*] [XX; 3 haploid sets; *intersex*]

Phenocopy:

an individual showing features characteristic of a genotype other than its own, but produced environmentally rather than genetically. Example: Vestigial wings in Drosophila are produced by a recessive mutation. This trait is also produced by high temperature during development.

Sex-influenced characteristic:

determined by autosomal genes and are inherited according to Mendelian principles, but are expressed differently in males and females.

Sex-limited characteristic:

encoded by autosomal genes that are expressed only in one sex; the trait has zero penetrance in the other sex.

If an XX individual has a defective androgen receptor what external characteristics would the person exhibit?

female bc androgen receptor is only present in males

Assume that long fingers are inherited as a recessive trait with 80% penetrance. Two people heterozygous for long fingers mate. What is the probability that their first child will have long fingers?

genotype: Ll x Ll= 1/4 ll (long fingers) phenotype: 80% of 1/4; therefore .80 x 1/4= answer: .2

The sex that produces two different types of gametes with respect to the sex chromosomes is called the _____ sex. dioecious monoecious pseudoautosomal heterogametic

heterogametic

Compound heterozygote:

individual that carries two different alleles at a locus resulting in a recessive phenotype. Example: cystic fibrosis

Traits encoded by genes located on the Y chromosome are termed Y-_____ traits. disjoined labeled dissociated linked

linked

In horses, the Overo gene, Ov, produces a white splotch pattern on the coat. The overo phenotype is seen only when a horse has one Ov copy, Ovov. Horses with two Ov copies, OvOv, die soon after birth and are called white overo because they are completely white. Horses with no Ov copies are solid colored, ovov. The Leopard complex gene, Lp, shows incomplete dominance and controls white spotting. One Lp allele, Lplp, produces the leopard phenotype, in which there are spots everywhere. Two Lp alleles, LpLp, produce the fewspot phenotype, in which the horse is mostly white with colored spots. A horse that is both overo and leopard is called pintaloosa, and these horses are spotted with splotches. A horse that is overo and fewspot is considered fewspot because the white areas from Lp is indistinguishable from the white from Ov. Suppose that 16 pairs of pintaloosa horses have one offspring per pair. How many of each phenotype would be expected? Determine the number out of 16 expected for each phenotype. Only count phenotypes for offspring expected to live past one week of age. - overo? - white overo? - fewspot?

overo: 2 white overo: 0 fewspot: 3

In biology, the sex (male or female) of an organism is defined in reference to its _____. presence or absence of a Y chromosome phenotype genotype presence or absence of two X chromosomes

phenotype

Maternal effect genes:

phenotype of individual is determined by genotype of mom.

Codominance:

phenotype of the heterozygote includes the phenotypes of both the homozygotes (This can only occur when there are more than two alleles present within a group of individuals.)

Complete dominance:

phenotype of the heterozygote is the same as the phenotype of one of the homozygotes

dioecious:

plant or invertebrate animal) having the male and female reproductive organs in separate individuals.

In the common fruit fly the presence of a Y does not make the animal male. Why?

the common fruit fly depends on the X:A ratio to predict the sex of the animal

Expressivity:

the degree to which a character is expressed. Example: the extra finger may be fully functional or it may be a small tag of extra skin

Incomplete dominance:

the heterozygote is a third phenotype that is between (falls within the range) the two homozygous phenotypes

Penetrance:

the percentage of individuals having a particular genotype that express the expected phenotype. Example - 90% of people with a gene for an extra finger may have the phenotype

XX-XY system:

•Female: XX •Male: XY (heterogametic) •many insects, fish, amphibians, reptiles, mammals, humans

ZZ-ZW system:

•Females: ZW (heterogametic) •Males: ZZ •butterflies, birds, some reptiles and amphibians

Sex-linked characteristics:

•determined by genes on the sex chromosomes •the sex of the parent and child matters for the phenotype and genotype •Ex: X-linked white eye in Drosophila •Ex: X-linked color blindness in humans

XX-XO system:

•females: XX •Males: X; heterogametic sex •some grasshoppers and other insects

Genic Sex determination system:

•no distinct sex chromosomes •sex determined by genes on undifferentiated chromosomes •the heterogametic sex will vary •occurs in some plants, fungi, protozoans, and fish

Environmental sex determination:

•sex determined by environmental factors •no heterogametic sex •some invertebrates •turtles: warm incubation temps produce more females •alligators: the reverse of turtles

SRY gene:

•the gene is y linked because is it found only on the Y chromosome •necessary to begin male development SRY promotes testes formation, which produces testosterone •Testosterone stimulates other tissues to develop as male through the androgen receptor


Related study sets

Designing an Accessible User Experience

View Set

HDFS OSU 2200 Midterm 1, HDFS 2200 Final OSU

View Set

Security+ 601 - Ch 14: Incident Response

View Set